Você está na página 1de 60

SEARA da CIÊNCIA

Antigas perguntas de Física.


Para novas perguntas e respostas, visite a SEARA DA CIÊNCIA.

PERGUNTAS RESPONDIDAS.
Clique sobre o R após uma pergunta para ler a resposta.
Os links nas respostas estão quebrados e não devem ser acessados.

Por que o leite derrama quando ferve e a água não?   R

Porque que a água no filtro de barro é mais fria do que na temperatura ambiente?   R

Sobre a resposta da pergunta acima, existe um comentario sobre resfriamento evaporativo.


Tambem co-responsavel pela obtencao de gelo no deserto (Antigo Egito). Ou voces acham que os
escravos jogavam ar quente nos faraos quando os abanavam?. Gostaria de saber mais sobre isso
dos egipcios fazerem gelo no deserto, se faziam.   R

Temos na Terra dois corpos, um maior que o outro, em uma balança de braços equilibrados na
posiçäo horizontal.O que aconteceria com a balança se levassemos essa experiencia para
lua?   R

Como varia a pressão atmosférica com a temperatura do ar?    R

O que provoca a desaceleração da luz quando ela diminui a velocidade ao passar do ar para água
? O que faz a luz aumentar a sua velocidade ao voltar para o ar ?    R

Por que a água no topo de uma montanha de 1000 metros de altura, por exemplo, ferve a uma
temperatura mais baixa que ao nível do mar?   R

Como atua a pressão atmosférica no momento em que tomamos um refrigerante através de um


canudinho?    R

Afinal de contas, a "força" centrífuga é ou não é uma força? E por que existem autores que não a
definem como força?   R

Porque a cerveja quando está na geladeira não congela, mas congela quando tocamos ao abrir a
garrafa?    R

Pode o som se propagar num feixe luz?   R

Se a energia de um corpo é função unicamente da massa (E = mc2), qual é a diferença, em


termos de quantidade de energia, entre 100g de açúcar e 100g de Plutônio?   R

Segundo a Lei da Gravitação Universal, "matéria atrai matéria". Ora, as singularidades dos
buracos negros têm massa quase que infinita(em relação a outros corpos) logo os buracos
negros deveriam atrair uns aos outros formando um buraco negro gigantesco cuja singularidade
era a soma das demais. Se tal acontecesse, todo o universo deveria ser "engolido" pelo buraco
negro em questão, situação tão dramática que, se fosse possível, cientistas já teriam detectado,
logo, tal não ocorre. Por que?   R

Se, conforme afirma a Teoria da Relatividade, matéria deforma o espaço-tempo e, conforme


afirma a Lei da Gravitação Universal, "matéria atrai matéria", toda a matéria do universo se
concentraria em uma determinada região do mesmo conferindo ao espaço-tempo uma forma
hiperbólica. Entretanto, tal não ocorre. Por que?   R
Explique porque um homem invisível deveria ser deveria ser totalmente cego.   R

A Lua é atraida pela Terra? Porque ela nao cai na Terra?   R

Qual a diferença ente gás e vapor?   R

Quando enchemos, com água, uma pia e, em segida, deixamos esvaziar vemos que surge um
movimento de rotação da água e que, para um mesmo local, o sentido é o mesmo. Por que isso
acontece? Há lugar onde isso não aconteça? Há lugares onde o sentido da rotação da água será
contrário?   R

Porque a água dos mares, rios e lagos começa a congelar por cima e embaixo não se
congela?   R

Já notei várias vezes que quando aparece um arco-íris, logo acima dele aparece um outro, só que
com as cores invertidas. Como isso acontece??   R

Temos noções intuitivas de carga e massa, mas na realidade nunca vi o que realmente elas são.
O que é massa? E carga elétrica?   R

É verdade que se ultrapassarmos a velocidade da luz, poderemos voltar no tempo?   R

Qual é o papel da ponte de hidrogenio no comportamento anormal da agua (densidade)???   R

Por que se deriva o hamiltoniano, já que ele representa a energia total, e pela lei da conservação
da energia ela se mantem constante? Derivar uma constante resulta em 0.   R

Como um corpo que possui absorvidade e emissividade iguais ,segundo Kirchhoff,pode aumentar
sua temperatura?   R

Alguém pode explicar sucinta e claramente como funcionam os buracos brancos?   R

Por que ocorre interferência na imagem da TV quando uma furadeira está ligada?   R

O que são táquions?   R

Existe alguma restrição em igualar E = mc2 e E = h.f? Existe alguma expressão que foi obtida e
que seja satisfatória igualando energias que se relacionam?   R

Tirar foto de outras galaxias, tudo bem. Mas, como se conseque tirar foto de toda nossa galaxia,
mostrando ainda onde fica nosso sistema solar, sendo que nao podemos sair dela?   R

Gostaria de saber como ocorre uma descarga elétrica (raio, relâmpago). Se uma nuvem está
carregada com cargas positivas, os elétrons sobem para neutralizá-la?   R

O princípio da incerteza de Heizenberg serve apenas para partículas microscópicas? Por que?   R

O que aconteceria comigo (e outras coisas) se eu caísse dentro de um buraco negro?   R

Quando seguramos algum objeto nas mãos logo logo nos cansamos de segurá-los. Por que, já
que esse objeto não está realizando nenhum trabalho?? Será que tem haver com a energia
potencial que ele possui?   R

O que é o movimento de "nutação" ? Porque o giroscópio não cai de uma vez por todas ao invés
de ficar descendo e subindo!!??   R

O que realmente está ocorrendo com o Sol? Está esfriando e, portanto, diminuindo de tamanho,
fazendo com que sua gravidade aumente e com isso atraindo mais fortemente os planetas ou
está aumentando de tamanho que num futuro irá engolir os planetas?   R

Uma pessoa exposta por 10 horas aos raios infra-vermelhos emitidos por uma filmadora noturna
corre risco de saude?   R
Por que as estações de lançamento de foguetes geralmente se localizam em pontos com
latitudes próximas á linha do equador?   R

O que é computação quântica e qual é a sua função??   R

Por que a velocidade da luz é representada pela letra c?   R

Existe alguma interpretação relativística ou quântica da Entropia?   R

Estava lendo a explicação sobre a imersão de corpos em fluidos (tora de madeira ou prego), e
porque uns flutuam e outros não, tudo é por causa da densidade.. (essa eu sabia), mas lendo
sobre as tempestades, vi o desenho da nuvem cumulus nimbus e as temperaturas associadas e
me perguntei: Porque que essa nuvem não desaba sobre a terra? visto que a água lá em cima tá a
uns -20ºC, isso quer dizer que está congelada, e gelo é mais denso que ar (que eu saiba), gostaria
de uma resposta satisfatória. Obrigado.   R

Em quais circuntâncias um próton e um elétron podem se repelir!?   R

Ao colocarmos uma toalha molhada no varal, em um dia de Sol, ela poderá secar completamnete,
assim como se colocarmos em uma sala com ar-condicionado. Por que? Nos dois casos, se
considerarmos o mesmo istante, qual secará primeiro?    R

O que é a barreira do som?   R

Se a temperatura do nosso corpo é de 36,5 graus celsius, porque sentimos tanto calor quando a
temperatura ambiente é 30 graus?   R

Gostaria de saber o que é um referencial não Inercial, a diferença entre referenciais não inerciais
e inerciais e alguns exemplos com algumas experiências!!   R

É sabido que as moléculas de som podem destruir um corpo sólido. Na bíblia consta no livro de
Josué que as muralhas de Jericó foram destruidas após o toque das cornetas. É possível que o
mesmo efeito do som em uma taça de cristal possa ter sido o fator que derrubou as
muralhas?   R

Segundo a teoria da relatividade restrita de Einstein, M = Mo/Sqrt(1-v^2/c^2), sendo Mo a massa


própria de um móvel. Para v>c (táquions) temos M (massa relativistica) complexa! Então, sobre o
que se fundamenta a afirmação que táquions tem massa negativa?   R

Gostaria de saber se ocorre aniquilação no contato de um neutrino com um antineutrino, já que


ambos têm carga nula.   R

Li no jornal sobre uma teoria que diz que se uma pessoa está parada no vácuo sem matéria, não
vê nada. Porém, se está em movimento nesse mesmo vácuo, vê partículas, como se fosse uma
chuva. Gostaria de saber melhor sobre essa teoria e como foi a experiência que uns brasileiros
fizeram e acabaram comprovando-a.   R

A Terra atrai para o seu centro todos os corpos próximos à sua superfície. Esta força é descrita
pela expressão que representa a lei da gravitação universal de Newton. Quanto maior a massa do
corpo maior será a força de atração gravitacional, é verdade? Se for verdade, então por que um
corpo de maior massa não cai mais depressa que um corpo de menor massa?   R

Como podemos mensurar o peso de um balão contendo gás Hélio, numa balança convencional,
num local que não seja o vácuo?   R

Ouvi dizer que existe uma generalização da Relatividade Geral de Einstein chamada "Gravitação
Não-Simétrica" que não apresenta soluções "anomalas" (i.e. buracos negros, wormholes, curvas
fechadas do tempo, etc...). Gostaria de saber se tal teoria invalida os resultados obtidos por
Hawking e qual é o seu impacto sobre a física moderna.   R

Porque é que um fotão anda à velocidade da luz?   R

Um buraco negro, após ter absorvido energia suficiente, volta a ser uma estrela?   R
Se antimatéria possui "densidade de energia negativa" porque ela não é repelida
gravitacionalmente?   R

Gostaria de saber se existe uma relação entre carga, energia e/ou massa, de modo a unificá-las
em uma só propriedade.   R

Afinal de contas os corpos caem na terra, por exemplo, porque há uma força de atração entre
eles e a terra ou porque a terra encurva o espaço em torno de si?   R

Eu gostaria de saber o que é um gráviton...Se existe, ele é uma partícula ou uma forma de onda?
A função do gráviton é atrair a matéria (ou outros grávitons), ou é deformar o tempo-espaço?   R

Na natureza nada se cria, nada se perde, tudo se transforma", e dizem que os buracos negros
"sugam" qualquer forma de energia e matéria. Onde vai parar esta energia ou matéria atraida pelo
buraco negro? Obrigado.   R

Em que princípio físico se baseia o velocímetro de um avião?, isto é, como um avião mede a sua
velocidade quando está voando?   R

Gostaria de saber como funciona uma antena parabolica? Teria como eu montar minha propria
antena parabolica?   R

Por que as bombas atômicas quando explodem têm sempre o formato de um cogumelo?
Obrigado.   R

Algumas pessoas (muitas delas tolmente leigas em física) acham que o gelo derrete mais
rapidamente dentro de água(à temperatura ambiente ) que fora dela . Isso é verdade? Caso seja
qual a explicação física para esse fenômeno?   R

A base do eletromagnetismo, nos diz que o campo magnético advem do movimento de cargas
elétricas. Se hipoteticamente estivermos "viajando" junto a carga, não existirá campo magnético.
O campo magnético depende do referencial?   R

Por que surgem bolhas ao agitarmos um recipiente com água que acaba de sair do
microondas?   R

Além da força de atrito, porque é difícil caminhar sobre areia (por exemplo, areia de praia)?   R

Um automóvel com pneus mais largos : -Trava melhor ? -O consumo de combustivel aumenta? -
Atinge maior velocidade? -É mais estável?   R

O vento pode distorcer uma onda sonora ao ar livre? Ou ainda, seria correto dizer que o vento está
levando o som em tal sentido?   R

O que é Simetria de Gauge? Por que devem existir partículas para mediar as forças
fundamentais? Essas partículas são "fundamentais"? O que são os diagramas de Feynmam?   R

O big bang se originou de apenas uma singularidade ou pode ter acontecido com mais de
uma?   R

Alguém poderia me explicar de forma simplificada o que é o espaço-tempo?   R

Por que em uma panela de pressão os alimentos cozinham mais rápido?   R

Como é que se utiliza a força da água nas barragens para obtermos energia hidro-eléctrica?
Qualquer rio (caudal) é suficiente para produzir energia ou tem de obedecer a certos
critérios/valores? No caso do ar (vento), utiliza-se o mesmo método nos moinhos?   R

Por que se unirmos duas lâminas de vidro de um microscópio fica muito difícil de separá-las?   R

Por que em alguns lugares a água dos oceanos é verde , noutros azul? O que caracteriza a cor da
áqua dos oceanos?   R

O que vem a ser o Mar de Dirac?   R


Por que as estrelas cintilam no céu enquanto os planetas têm brilho fixo?   R

Se bronzeamento da pele é causado pela exposição a raios infravermelhos que conseguem


atravessar as nuvens,por que as pessoas de países europeus(frios)são em sua maioria
brancas?   R

Um avião leva o mesmo tempo para contornar a Terra independente do sentido(leste-oeste ou


oeste-leste)?   R

Se o Sol exerce na Terra uma força 175 vezes maior do que a que a Lua exerce na Terra, então
porque é a Lua que causa o fenômeno das marés e não o Sol?   R

Como são formadas as cores em um aparelho de TV?   R

Todas as estrelas que vemos no céu pertencem á Via-Láctea?   R

Gostaria de saber se é possível construir um dispositivo que possa anular a força da gravidade.
Como ele seria?   R

Quando duas cargas elétricas puntiformes e em repouso estão em meios distintos como é
determinada a constante eletrostrática para efeito do cálculo da força elétrica?   R

Ouvi dizer que Einstein disse que o espaço é curvo. Ele conseguiu provar? Como?   R

Qual é forma geométrica e aparência(esferas, cubos, poliedros etc.) das partículas, como
elétrons, prótons, neutróns etc.? Se ainda não é possível determinar as formas dessas tais
partículas seria possível determiná-las através de cálculos matemáticos? Será possível, algum
dia, observar tais partículas através de instrumentos sofisticados? Um abraço.   R

Como funciona uma câmara de bolhas? Por que ela tem esse nome?   R

Imaginemos uma nave espacial na velocidade máxima (a da luz). Se alguém dentro dela se
deslocar na direção da trajetória (p/ frente) esta pessoa não ultrapassaria a velocidade da luz? O
q aconteceria?   R

Quais são os melhores condutores eléctricos e porquê?   R

Por que nos estados do nordeste, a tensão é de 220V, enquanto nos estados do sul ela é apenas
110V?   R

Está correta a afirmação de que se apertarmos uma garrafa de plástico de refrigerante antes de
fechá-la novamente e deixarmos menos espaço para o gás sair do líquido isso fará com que o
refrigerante perca menos gás?   R

Existe uma pequena controvérsia entre professores de cursinhos pré-vestibulares de Brasília. Uns
dizem que buracos-negros existem e contam com comprovação científica. Outros dizem que isso
é besteira e que eles não existem. Em qual posso confiar?   R

Se o universo se expande de onde vem a energia para isso?   R

Por favor, me ajudem!!! O clássico problema de uma ampulheta em cima de uma balança... no
instante em que um grão da ampulheta cai, sem estar em contato com esta, a balança mudará
sua marcação? sim, não?Por que? E depois q bater no piso da ampulheta?   R

O que é o Método de Monte Carlo e quais são as suas aplicações em física (e fora dela)?   R

Como que algo proximo à velocidade da luz pode interferir no tempo?   R

Se a força da gravidade em nosso planeta ficasse de repente menor, as pessoas que ainda estão
crescendo teriam uma altura maior que aquelas que cresceram com a gravidade que temos
hoje?   R

Oi, gostaria de saber em que estado da matéria se encontra a fumaça.   R


Se no espaço há bilhões e bilhões de estrelas dentro de galáxias e que por sua vez há bilhões de
galáxias então por que o espaço é basicamente uma escuridão?   R

Como se pode afirmar com tanta certeza que a quantidade de matéria no universo é maior do
que a de antimatéria?   R

Se um foton cai em uma singularidade, eu poderia saber seu spin verificando o seu anti-foton que
está fora da singularidade? Ocorreria o efeito de teleporte de informação do foton para o seu anti-
foton?   R

A radiação tem massa?   R

É verdade que, se todos os chineses do mundo andassem na mesma hora, para o mesmo lado, a
velocidade de rotação da Terra mudaria? Que efeitos isso poderia trazer?   R

Li neste site que, se púdessemos ultrapassar a velocidade da luz, voltaríamos no tempo.


Abstraindo o fato de que viraríamos energia ao alcançar tal velocidade, gostaria de saber: isto
significa que o passado está registrado fisicamente em algum lugar? E caso esteja, ele poderia
ser alterado?   R

As linhas de forças que saem de uma carga elétrica positiva implicam na existência de uma
carga negativa que as receba?   R

Como se chama o quinto estado da matéria, e para que serve?   R

O que é precessão de equinócio? E como funciona?   R

Se todas as galáxias estão se afastando uma das outras, então por que a galáxia de Andrômeda
está se aproximando da Via-Láctea?   R

O que é o Tempo de Planck? Ele é o exato momento do Big Bang?   R

Por que quando estamos parados(de carro) num sinal e um outro carro lentamente passa ao
nosso lado , temos a sensação de que o nosso carro está se movendo?   R

Numa cobrança de penalti, quantos segundos se passam? E quanto tempo o goleiro tem para
escolher o canto da bola?   R

Gostaria de saber o que são membranas com 10 dimensões e supergravidade com 11 dimensões
e se elas tem algo em comum. Obrigado!   R

Qual a aplicação prática da faixa de Mobius? Existe alguma equação que a possa definir
analíticamente?   R

É possivel fazer uma maquina que possa filtrar a água para fazer oxigenio (tirar o hidrogenio e
outras coisas sobrando o oxigenio)?   R

O que é "quantum loop gravity"? Há simulações numéricas sobre esse tema?   R

A teoria diz que uma onda não pode transportar matéria... mas como é possível surfar?   R

Sou proprietária de um apto no 1º andar de um prédio que foi "escolhido" por uma empresa de
telefonia celular para que seja instalada no terraço a sua antena, mediante um aluguel mensal e
por 10 anos. A princípio sou contra pois acho que posso sofrer irradiação... Pode me esclarecer?
Muito grata.   R

A Magnetita, sendo um ímã natural, pode ser desmagnetizada?   R

Se o hélio é menos denso que o ar, e o som se propaga mais rápido em meios mais densos, por
que então a velocidade do som no hélio é 965m/s e a velocidade do som no ar é 343m/s?   R

Gostaria de saber a respeito do Demônio de Maxwell.   R

Como calcular o campo magnético da Terra? E através de quais experiências?   R


Gostaria de saber sobre a história da mulher na física. Obrigada!   R

Como foi que Avogadro chegou ao seu famoso numero?   R

Quando está muito frio, sua pele fica arrepiada. Por quê?   R

Por que quanto mais fundo mais azul fica a agua do mar?   R

Li em um livro que a antimatéria não segue no tempo igual a matéria, ou seja, ela segue do futuro
em direção ao passado. O livro diz que um elétron voltando no tempo tem o mesmo efeito que
um positron avançando, pois está sendo eliminada uma carga negativa do futuro. Isso é verdade?
É comprovado?   R

O quê significa (e como se obtém) Watts RMS e Watts PMPO? Existe alguma relação matemática
entre os dois? Notei que aparelhos de som de fabricantes diferentes nem sempre apresentam a
mesma relação.   R

É possível que ondas cerebrais interfiram na matéria e/ou em outro cérebro? Como?   R

Por que o congelador fica em cima da geladeira?   R

O que é o paradoxo dos gêmeos?   R

Por que mesmo o sabonete tendo cores variadas a espuma sempre é branca?   R

Como funcionam aquelas placas coletoras de energia solar que algumas casas têm em seu
telhado?  R

Desejo inflar um balão com gás hélio e amarrá-lo a um objeto com X gramas. Como faço para
calcular o volume de gás hélio necessário para flutuar esse objeto no ar?   R

Qual o motivo do mês de fevereiro ter 28 dias?   R

No livro O Universo Numa Casca de Noz, Stephen Hawking diz que os táquions são partículas
cujo quadrado da massa é negativo. Como isso é possível?   R

Além dos grávitons, existem outras partículas subatômicas que são previstas mas ainda não
foram descobertas? Quais são elas?   R

Será que com um campo electrico forte podemos parar um electrão? Se sim, poderiamos saber
com exactidão a posição onde a sua velocidade se anula?   R

Se existisse um helicóptero com autonomia de 24 hs, e ficasse parado na vertical durante esse
período, seria possível ele dar a volta ao mundo?   R

Que consequências poderia haver se alguem tentasse recriar um big bang, ou algo parecido,
dentro da Terra?   R

Qualquer corpo ou particula dotado de momento angular tem um efeito rotativo?   R

O que é o vento solar?   R

Qual foi a primeira tentativa para descobrir a velocidade do som?   R

Como foram descobertos os neutrinos? É certo que eles existam?   R

Por que as crianças sentem mais frio que os adultos?   R

O que são ondas gravitacionais?   R

Por que a serragem é um melhor isolante térmico do que a madeira?   R

Por que usamos a escala Kelvin (S.I.) na Lei Geral dos gases?   R

Por que nos fios de alta tensão tem aquelas bolas cor de laranja?   R
Que partículas constituem o átomo, além dos prótons, elétrons e neutrons?   R

O que é o efeito piezo-elétrico?   R

O que existe no vácuo entre as estrelas?  R

As ondas do mar que chegam até a praia, vão diminuindo sua velocidade e em seguida são
puxadas para trás . Que forças são essas que sugam a água de volta?   R

O que são redes neurais? Elas tem alguma relação com a mecânica estatística?   R

O aspecto do céu noturno, observado de um mesmo local numa mesma hora, muda um pouco a
cada dia, mas ao final de um ano, volta a se repetir?   R

Como uma nave espacial se desloca no espaço? ( de um lugar para outro, tipo, da terra para
marte?)   R

Como os cientistas sabem a temperatura das estrelas?   R

A televisão emite raios-X?   R

Peguei uma folha de papel, e fui dobrando ao meio e contando, e só consegui dobrá-la oito vezes,
depois tentei com uma pedaço maior e novamente só oito vezes, quero uma explicação física
para isso?   R

O que é Referencial Nao-Inercial?  R

Qual o fenômeno físico que explica a ocorrência de listas periódicas se movimentando nos
monitores de computador quando filmados por câmeras de vídeo?   R

Por que a soma da massa dos quarks uud e udd é centenas de vezes inferior à massa do proton e
do neutron?   R

É possível fotografar átomos? Onde consigo fotos desse tipo?   R

Como funciona a panela de pressão? E quem inventou? Obrigada!   R

Gostaria de uma explicação a respeito da relação entre Precessão dos Equinócios e a Declinação
Magnética. Porque ela varia no tempo e no espaço?   R

Se colocarmos um pósitron ao lado de umpróton eles irão se atrair (já que são matéria e
antimatéria) ou se repelir (já que têm a mesma carga)?   R

Por que as veias são vistas azuladas se o sangue é vermelho?   R

Um corpo eletricamente neutro pode atrair a outro? Como um átomo sendo neutro pode se
prender eletricamente a outro(s) formando moléculas?  R

Se há necessidade de oxigênio como comburente para produzir fogo, podemos então afirmar que
uma arma de fogo não pode ser disparada no espaço?   R

Por que nao podemos reconhecer a nossa voz gravada numa fita cassete?   R

Por que uma das galáxias do Universo se chama Via Lactea?   R

Quanto % tem no nosso corpo de água?   R

Como medir de maneira simples e com pouca margem de erro o volume do ar expirado pelos
pulmões, sem recorrer a tecnologias despendiosas?   R

O que significa lente polarizada, termo este utilizado usualmente quando se refere a óculos
escuros? E é verdade que quando usamos óculos com lente amarela de noite para dirigir carro
enxergamos os objetos de forma mais nítida?   R
Se uma partícula de matéria fosse colocada em contato com uma partícula de antimatéria que
não é a sua correspondente haveria aniquilação total da partícula de menor massa?   R

O que significa fazer com que a constante de Planck seja igual a zero? O que aconteceria?   R

Quais os efeitos da ausencia da gravidade no corpo humano?   R

Gostaria de saber se uma pessoa conseguiria tomar um refrigerante com canudinho na lua como
se estivesse na Terra.   R

O que é efeito Peltier e como ele age em junções semicondutoras para resfriar e aquecer
liquidos?    R

Como é calculada a distância da Terra à uma galáxia?   R

O desvio para o vermelho de linhas espectrais de estrelas, galaxias, é falado que é causado pelo
efeito doppler da luz. Mas existe comprovacao desse efeito em laboratorio?   R

Eu gostaria de saber se o uso de produtos que emitem raios infra-vermelho longos- aqueles
utilizados em produtos para emagrecer eeliminar gordura podem causar algum dano à saúde?   R

Gostaria de saber um pouco sobre os spins e como são aplicados na ressonância magnética.   R

O que é Holografia? Qual sua relação com a Física Ótica?   R

O que é cristal líquido? como funciona? e onde ele é usado?   R

Porque é tão dificil para os físicos acreditarem que Deus é Criador do Universo?   R

É verdade que se ocorrer um acidente grave numa usina nuclear o núcleo pode até atravessar o
piso da usina e adentrar ao solo até atingir o centro da terra? E se isso for verdade o que ocorreria
em seguida? Uma erupção de um vulcão com lava radioativa?   R

Todos sabem que abaixo de 0ºC a água (H2O) se congela. Li em uma reportagem séria (revista
Superinteressante) que se você for baixando a temperatura, de -122ºC até -135ºC ela volta a ser
líquida, e de -135ºC para baixo volta a se solidificar. Porque isso acontece?   R

A respeito de som, li em uma revista (a Super de novo, hehe), que conseguiram dar "foco" ao som
como se fosse uma lanterna, podendo ser direcionado, sem que quem não está na frente não
consegue ouvir (excluindo alguma reflexão pelo chão ou parede). Se o som é a propagação das
ondas na matéria, como é possível direcionar o som se a fonte do mesmo está em contato com o
ar e ele propagaria as ondas em todas as direções?   R

É possivel magnetizar a borracha?   R

Qual a temperatura que o CO2 se liquefaz?  R

Suponha que os valores das cargas nos elétrons e nos prótons fossem diferentes entre si. O
mundo seria diferente? Obrigado!   R

Qual é a densidade de um buraco negro?   R

O que é um relampago esférico?   R

O que é e o que pode impedir a formação daquele fungo verde nos potes de barro?   R

Como os esquimós constroem os iglus?   R

Quando o ar é mais rarefeito o som se propaga em uma velocidade maior?   R

Como a anti-matéria poderia ser utilizada de forma benéfica a ciência? Poderia a anti-matéria ser
utilizada para causar danos assim como a bomba atômica?   R

Gostaria de saber ao assoprar uma vela, porque ela apaga?   R


Os bósons são matéria (têm massa e ocupam lugar no espaço)?   R

O que aconteceria se o núcleo e o manto da Terra fossem resfriados?   R

Uma carga Q está caindo com aceleração g. Ela irradia? Se irradia, de onde vem a energia da
radiação?   R

Por que os satélites dos planetas gasosos são rochosos,se na época da formação do Sistema
Solar a matéria da nebulosa que os originou era a mesma?   R

É verdade que existe mais de um universo?   R

Quando afirmamos que a matéria deforma o espaço-tempo, significa que ela deforma o espaço.
E o tempo? Como seria essa deformação no tempo?   R

Como Newton explicou a ação a distância (força) e como a terra 'sabe' que existe alguma corpo
nas suas proximidades para ser atraído?   R

Existe uma lei de conservação de spin?   R

Se assumirmos a segunda lei da termodinâmica, ou seja, a lei da Entropia, devemos considerar


que Charles Darwin estava errado. Afinal, a evolução contradiz a Entropia. Então a pergunta é a
seguinte: Como relacionar a segunda lei da termodinâmica à Evolução?   R

O que é uma máquina de moto-contínuo,e por que elas não podem existir?  R

Quais são os processos físicos envolvidos na formação de um tornado e qual é a diferença entre
tornado e furacão?  R

Ouvi falar que a teoria dos quarks é capaz de provar que a teoria do evolucionismo está errada.
Isso é verdade?  R

Por que não devem ser usadas lâmpadas verdes ou vermelhas para iluminar balcões onde serão
expostas carnes nos açougues?  R

O que vem a ser "velocidade de dobra"?  R

Além do Big Bang, existe alguma outra explicação científica para a origem do universo?  R

Porque é que se vê uma espécie de nuvem, em forma de anel, à volta do Concorde quando este
atravessa a barreira do som?  R

Por que os oceanos e mares se tornaram salgados?  R

Existe diferença de orientação entre as Bactérias Magnéticas que se situam próximas ao polo sul
magnético, polo norte magnético e equador?  R

Gostaria de saber, baseado no conceito de hidrodinâmica, como funciona o aparelho de sugar


usados nas cozinhas para sugar o vapor que sai das panelas e utensílios sobre o fogão.
Obrigada!  R

Por favor, expliquem de forma a tornar possível uma visualização. Quanto às supercordas, o que
significa dizer "cordas abertas" e "fechadas"? Qual a diferença física? e quanto aos modos de
vibração? Se referem mesmo á uma frequência real? Como as diferentes frequências podem
originar partículas diferentes se a natureza das cordas é a mesma? Obrigado.  R

Ontem, na televisão, vi notícias acerca do alinhamento de planetas que ocorre a cada 40 anos, e
vários comentários, sendo uns científicos e outros voltados para crenças. Alguém pode me
orientar sobre como obter mais informações a respeito? Agradeço.  R

Nós seres humanos produzimos ondas (sonoras), correto? seria possivel alguém produzir uma
frequencia de onda que fosse iqual a frequência de um eletrodoméstico (controle remoto da tv
por exemplo)?  R
Por que com o passar dos anos os homens (masculino) perdem a audicao para sons de alta
frequencia?  R

Gostaria de saber como se forma a chuva de granizo?  R

Por que a tinta da caneta não sai pela abertura de trás?  R

O que faz com que alguns objetos tenham a capacidade de brilhar no escuro?  R

Quais processos utilizamos para separar um pouco de sal e um pouco de açucar misturados?  R

Qual a diferença no preparo de um alimento feito no fogo a gás ou a lenha, para o alimento feito
ao microondas?  R

Tenho observado nas partidas de futebol pela TV que anúncios (propagandas)colocados atrás da
linha de fundo do campo, nos dão a nítida impressão de que estão posicionados na vertical e no
entanto estão estendidos no chão, ou seja, na horizontal. Como se explica esse efeito visual?  R

Fala-se muito aqui em Radiação de Hawking, que seria a radiação emitida pelos buracos negros.
Certo... Mas no livro O Universo em Uma Casca de Noz, Hawking diz que o buraco negro não
emite nada... O que acontece é que existem tipos de particulas que vivem em pares, se anulando
a cada momento. Quando um par desses passa ao lado de um BN, ele suga uma das particulas e
a outra não consegue se anular, parecendo, entao, que foi expelida... Ah! Que zona! Alguém me
explica o que acontece?  R

Cite qualquer questao que alguns veem como paranormal e a fisica explica. Por favor como
explicar o caso da ladeira do amendoim em Belo Horizonte- MG.?  R

Gostaria de saber algo sobre a flecha do tempo e a irreversibilidade, bem como o que significa
dizer que o tempo surgiu antes da matéria. (Entrei em contato com a matéria no livro de Ilya
Prigogine - O fim das certezas).  R

Se os gravitons propagam-se com a mesma velocidade dos fotons, como podem escapar do
evento de horizonte de um buraco negro?  R

Gostaria de saber se a velocidade de passagem da areia numa ampulheta é constante?  R

Eu sei o que é a barreira do som e como é que é provocada. Só não entendo o que é que a torna
visível (ou se é mesmo visível). Será devido a condensação da matéria provocada pela grande
pressão a que foi sujeita? Obrigada e desculpem a insistência.  R

Gostaria de saber mais sobre as p-branas. Como se comportariam essas p-branas? Seriam
também formada de minúsculas cordas? Tudo que se relaciona ao assunto me interessa...
Grato!  R

Ah... Outra questão de Física Teórica. O que seria a Teoria da Supergravidade (pra leigos, please!
:) e porque ela foi descartada? Obrigado!  R

A densidade do universo está diminuindo à medida que ele se expande? Quais são os efeitos
desse fato?  R

Não existe efeito sem causa. Sendo assim, como pode ter acontecido o Big Bang, se antes não
havia nada?  R

Por que o homem que anda sobre a corda bamba , quando está parado sobre ela não fica
estático , mas sim movendo-a para a esquerda e para a direita com o(s) pé(s)?  R

Quando, por acidente, encostamos o cotovelo em algum lugar sentimos uma sensação de
choque elétrico. Gostaria de saber se está correto afirmar que isto é um choque elétrico. E, se for,
qual a intensidade da corrente elétrica?  R

Hoje sabemos, por exemplo, que a água é formada de dois hidrogênios e um oxigênio e que o gás
carbônico é formado de dois oxigênios e um carbono. No entanto, como conseguiram descobrir
isso, se a tecnologia era tão elementar no passado?  R
Quando os satélites caem eles causam riscos para as pessoas que moram nos lugares prováveis
de queda?  R

Gostaria de saber quais os combustiveis alternativos q teriamos se o petroleo acabasse hoje?  R

Sobre a temperatura de cor de corpos irradiantes, se utiliza uma fórmula que usa comprimento de
onda para achar a temperatura (fórmula de Wien). Assim se acha, por exemplo, a temperatura
externa do sol (6000k). Mas, sem ser essa radiação térmica, os corpos frios que emitem luz, não
se pode usar essa mesma lei para determinar sua temperatura, ou pode? O vagalume que emite
luz verde ou outra, não está a milhares de graus kelvin de temperatura.  R

Por que só em determinados ambientes como cavernas e grandes salões ouvimos o eco de
nossa voz?  R

Como é que as lentes de contato funcionam?  R

Como posso atingir o zero kelvin?  R

Sabe-se que as ondas eletromagnéticas viajam no vácuo a aproximadamente 300 000 km/s.
Sabe-se, também, que em outros meios materiais a velocidade delas é menor. Qual é o principal
fator responsável por essa redução de velocidade?  R

Queria saber sobre o cone futuro da luz. Obrigada!  R

Por favor gostaria de saber o nome das 32 direções da rosa-dos-ventos, bem como onde poderia
encontrar um desenho completo dela.  R

Gostaria de saber o porque das antenas ou detectores de ondas eletromagnéticas em geral


serem sensíveis ao Campo Elétrico (E) ao invés do Campo Magnético (B) ou ambos, grato a
quem puder ajudar.  R

Eu gostaria de saber se existe uma equação para hélices. Que fatores interferem na impulsão de
uma e tudo mais q vcs puderem me dizer.  R

Se todos os países que detêm potencial de bombas atômicas,nucleares e químicas quisessem


destruir seu arsenal jogando todas as bombas no sol ou em alguma estrela qualquer, o que
aconteceria?  R

Existe alguma forma de relacionar o coeficiente de dilatação de uma substância com seus
pontos de fusão e ebulição?  R

Em quais aspectos a física quântica não pode se relacionar com a relatividade?  R

Os astronautas que estão na nave Discovery no filme "2001, uma odisséia no espaço" não
flutuam, como frequentemente vemos na tv ou no cinema. É verdadeira essa situação?  R

Seria possível a fabricação de hidrogênio tendo como fonte de energia painéis solares e qual
seria a relação entre a área de painéis solares e a quantidade de hidrogênio fabricada? Gostaria
também de saber qual seria o consumo de uma turbina à gas alimentada com hidrogênio,em
megawatt/hora? Obrigado.  R

O tempo de viagem Terra-Marte é diferente da viagem Marte-Terra. Qual o motivo?  R

eu gostaria de saber se já existe no brasil algum orgão que trabalhe com levitação magnética, e
se é possível fazer qualquer metal levitar através da levitação magnética, ou se tem que ser um
material especial para isso.  R

Antigamente era comum a exibição de faquires, homens que jejuavam, cercados de serpentes,
deitados em camas de pregos com as pontas voltadas para cima. Por que esses faquires não se
machucavam quando estavam deitados nessas camas?  R

Por que os vidros do carro embaçam quando está chovendo?  R


Quanto à poluição eletromagnética, como interfere no organismo humano, como pode ser
diagnosticada e quais são os antídotos para combatê-la?  R

Por que quando coloca-se uma pilha no congelador ou na água fervendo é recarregada?  R

O que é o vácuo? Há entropia no vácuo? Existe energia no vácuo?  R

Os faróis de um automovel são apontados para a parede de uma garagem. Por que não
aparecem franjas de interferencia na região em que os feixes dos farois se superpoem?  R

Por que o átomo de Tecnécio (Z = 43) foi sintetizado em laboratório, sendo que todos os demais
elementos com número atômico menor que o do Urânio existem naturalmente em nosso
planeta?  R

Gostaria de saber quais as caracteristicas das ondas (amplitude, frequencia) são usadas para
telefones móveis, i.e., celular e/ou telefone sem fio. Grato.  R

Por que o torque ( momento de uma força) não possui como unidade o Joule(J), já que sua
definição possui o produto força x distância (Newton x metro)?  R

Como seria o comportamento magnético de um imã esférico maciço? E se ele fosse oco?  R

Gostaría de saber o princípio de funcionamento de colchões sustentados pelo ar (aqueles usados


em máquinas tridimensionais de medições) e hovercraft.  R

Para encontrar o número de massa eu devo equacionar: o número atômico mais o número de
neutrons. Mas, se eu não tenho o número de neutrons e só tenho o número atômico, como faço
para resolver este problema? Há alguma tabela aonde eu posso pesquisar?  R

Qual é a origem do movimento rotacional da Terra? Ele é inalterável no tempo? Obrigado.  R

Dizem que para verificarmos se um microondas está vazando micro-ondas basta colocar uma
maçã em cima do mesmo e ligá-lo. Se ao cortarmos a maçã e ela estiver escura, temos um
problema de vazamento. É possível ser dessa forma?  R

Por que costuma-se soprar sobre a superfície de um líquido quente para que ele resfrie mais
rapidamente?  R

O que são ondas gravitacionas? Em que meio elas se propagam? E quais são as semelhanças
com as ondas eletromagnéticas?  R

Que diferenças (clássicas e quânticas) existem entre um campo de radiação e um campo de


forças?  R

Eu queria saber se a força de atrito cinético pode aumentar a velocidade de um corpo? como?
Cite um exemplo.  R

Li que os metais, como o COBRE, são ótimos condutores de eletricidade e entendi o porque.
Fiquei pensando sobre a utilização do cobre para propagar som . Ele é um condutor de vibrações
sonoras? Se sim ou não, vcs poderiam me dizer por que?  R

Por que a água apaga o fogo?  R

Einstein formulou a teoria da relatividade em 1905, mas só recebeu o Nobel por ela em 1921;
uma teoria de De Broglie formulada em 1923 só recebeu o Nobel em 1929. Por que um cientista
só recebe o Nobel muitos anos depois que faz sua teoria?  R

Como os cientistas conseguiram provar que o éter não pode existir?  R

A curvatura do espaço-tempo afeta o universo quântico?  R

Essa discussão sobre o Principio da Incerteza... Uns dizem que ele é uma impossibilidade
matemática, outros dizem que é fisica... Afinal, quem está certo? Aquela história do eletron
interagir fortemente com a luz, sendo impossivel determinar sua posiçao precisa, não faz parte
da explicaçao do Principio da Incerteza? Alguém me clareia as idéias, por favor! Grato.  R

O Daniel falou em outro servidor sobre a teoria das supercordas, que admitem 26 dimensões, das
quais 16 são "compactificadas". Gostaria de saber como é esse processo de "compactificação" e
como foi possível fazer experimentos para aceitar a teoria das supercordas com 10 dimensões
como uma teoria fisica viavel. Grato!  R

Gostaria de saber a relação existente entre os harmônicos esféricos e o cálculo do potencial


gravitacional terrestre. Como saber, por exemplo, se um harmônico está indicando um
achatamento ou um inchaço?  R

Como funcionavam os cilindros de alumínio criados por Joseph Weber(1919) para detectar os
grávitons?  R

O próton se divide?  R

A Constante de Huble nos indica que galáxias mais distantes se afastam de nós a maiores
velocidade. Mas como tais avaliações se referem a tempos totalmente diferentes, não seria
correto dizer que quanto mais no passado maior a velocidade das galáxias?  R

O que sao vidros de spin, eles tem alguma relacao com as spin networks (de Roger Penrose)?  R

Por que a medida que mergulhamos no mar, todos os objetos tendem a ficar azulados antes de
vir a escuridão total?  R

Existem teorias tais como a das cordas, da deformação do tempo-espaço e outras que precisam
de muita boa vontade e fé para compreensão, tendo se em conta a pouca quantidade de indícios
concretos a seu favor. Desculpem me a pergunta, mas diante de tantos indícios da possibilidade
de nossa existência espiritual, por que a falta de estudos neste campo?  R

A expansão do universo provoca um aumento na sua temperatura?  R

Quando criança, até os 14 anos, mais ou menos, via todas as noites a Via Láctea no céu, agora
tenho procurado e não a vejo mais. Por que? Para onde ela ou nós fomos?  R

Pode o tempo ser quantizado assim como a energia?  R

De onde saiu toda essa matemática maluca que molda as teorias físicas? Ela simplesmente foi
sendo inventada... E fim de papo? Sei que parece uma pergunta estranha, mas estranho, pra mim,
é o fato de sairem inventando matemática e fazendo predições através dela! Grato!  R

Li uma resposta (do Daniel) que dizia que os Buracos Negros só aparecem na Relatividade Geral,
mas o "problema" dos BN's já tinha sido exposto em 1700, quando um professor questionou a
existência de um corpo com uma gravidade tão grande que a velocidade de escape desse corpo
seria igual ou superior à velocidade... Ou seja, os BN's parecem ser um problema "clássico."
Alguém pode comentar isso? Grato!  R

Gostaria de saber o que acontece com a massa molecular do gás hélio, ao ser ionizado, há
alguma alteração no seu peso molecular?  R

Li na Folha de S.Paulo, em uma matéria sobre buracos negros, que existem buracos negros que,
de acordo com alguns astrofísicos, estariam emitindo radiação em uma velocidade maior que a
da luz. Isso, de alguma forma, seria possível? Será que há um furo na teoria da relatividade?  R

Se tudo é relativo, a velocidade da luz é relativa?  R

Que tipo de lente terei que usar para montar um microscopio?  R

Os desenhos artísticos dos buracos negros sempre o mostram no formato de um funil (buraco).
Mas o correto não seria imaginá-lo como uma bola, tendo a singularidade no meio, como se
fosse um planeta negro? (considerando que ele não esteja girando, é claro).  R

O que é um parsec? Isso só existe em StarWars mesmo ou é uma unidade real?  R


Como funciona um acelerador de partículas e quantos desses temos aqui no Brasil?  R

Olá, já faz algum tempo que eu quebrei um tubo de imagem por engano. Gostaria de saber se o
material com o qual eu tive contato (tive que recolher os cacos) pode causar algum tipo de dano
ao meu organismo. Obrigado.  R

Porque nos pólos da Terra a temperatura é mais baixa?  R

Os físicos modernos ficam tentando juntar a Teoria Quântica à Relatividade Geral, pra quantizar a
gravidade. Mas será que a gravidade PODE ser quantizada? Segundo a Relatividade Geral, a
gravidade existe porque a massa/energia curva o espaço tempo, certo? Então, pra que seria
necessária uma partícula mediadora dessa interação, uma vez que a causa do efeito já foi
observada? Parece que gravidade e fenômenos (sub)atômicos são meio diferentes entre si, não-
conciliáveis. Alguém comenta?  R

No filme 2001, uma odisséia no espaço, de Stanley Kubrick, tem a cena em o astronauta Franck
Poole vai consertar o exterior da nave. O computador o desconecta da nave e ele mergulha no
espaço. Pensei que por ele estar na mesma velociade da nave e por não haver atrito, ele
continuaria com a mesma velocidade, a mesma direção e sentido da mesma. Ou não?  R

Como faço para conectar um produto(220 Vac/50 Hz) em 60 Hz ? Existe algum tipo de conversor
ou outro equipamento?  R

Pra que serve o fio terra?  R

Eu fiquei sabendo que uma arma de fogo só dispara ,porque a pólvora contem oxigênio
próprio,sendo que em um projétil não há espaço para conter oxigênio. Em uma discussão,com
um colega de trabalho,o mesmo me disse sobre o filme ARMAGEDON ,que mente ao mostrar a
explosão de um asteróide no espaço,onde não existe oxigênio.Eu disse ao mesmo,que pode-se
sim obter a explosão pelo fato de os explosivos conterem polvora que é uma substância que
possue oxigênio próprio. A pergunta é :É VERDADE ESSAS INFORMAÇÕES MINHAS SOBRE A
PÓLVORA,OU EU ESTOU ERRADO,PASSANDO O MEU COLEGA DE TRABALHO A TER A RAZÃO?  R

Por que um tecido fica mais escuro quando jogamos água nele?  R

Qual é a menor partícula existente?  R

Se a pressão da água aumenta a temperatura, pq no fundo do mar a temperatura é baixa?  R

O sentido dos ponteiros do relógios obedece algum princípio científico? Poderia ser no sentido
contrário?  R

Olá! Sou técnico em segurança do trabalho e preciso saber maiores informações a respeito dos
riscos à saúde de trabalhadores expostos a radiação ultravioleta, proveniente de lâmpadas
utilizadas para desinfecção de garrafas, após a lavagem, em uma industria de bebidas. Gostaria
de saber, também, quais as formas de se impedir a propagação desta radiação, caso esta
exposição seja prejudicial aos trabalhadores deste setores.  R

Todo corpo/partícula acelerado à velocidade da luz necessariamente vira energia? Ou pode


acontecer apenas uma modificação na sua estrutura? Que fatores determinam essas duas
situações?  R

Qual foi a explicação que Gauss deu pro fato da força gravitacional ser inversamente
proporcional ao quadrado da distância?  R

É possível que um próton orbite um anti-próton, ou vice versa, assim como o elétron orbita o
próton?  R

O que é efeito Tunelamento? Efeito Túnel e Efeito Tunelamento são a mesma coisa?  R

O que é a super simetria?  R

Se a teoria da supersimetria estiver certa, por que as partículas simétricas às que conhecemos
ainda não foram identificadas em aceleradores de partículas?  R
Por que os planetas executam órbitas elipticas?  R

Qual a comparação entre gás ideal e gás real?  R

Será possivel que a natureza, e tambem o espaco tempo, grave as imagens das vidas das
pessoas, e com o passar dos anos, através de televisores ultra modernos, sera possivel verificar
estes registros, e no caso conheceremos a historia da humanidade?  R

Lendo algumas perguntas\respostas, acabei ficando confuso: um certo objeto, ou nós mesmos,
sofre que tipo de alteração na sua estrutura (eu digo: atómos, moleculas, massa, volume) a
velocidades maiores que c? E a velocidades maiores que 10c?  R

No filme de Ridley Scott -Alien, o Oitavo passageiro - quase no final, quando a tenente Ripley
numa pequena nave deixa a nave mãe Nostromo, ela â vê explodir (de longe). Fiquei surpreso
pelo fato de se ouvir o barulho da explosão, pois havia aprendido no ginásio que o som não se
propaga no vácuo. Agora estou confuso: quem está certo? Ridley Scott ou a minha professora de
ciências do ginásio (dona Maria Júlia)?  R

Como é que o tempo pode ser considerado um dimensão de acordo com a Relatividade Geral de
Einstein, se ele não pode ter uma medida como a da largura, comprimento e altura?  R

O que faz da água um bom condutor de eletricidade e por que ela perde essa característica
quando se encontra no estado sólido ( gelo )?  R

São válidas todas as inferências feitas através de fórmulas físicas? Ex.: fat = mi*N (força de
atrito). Por essa fórmula, podemos ver que quanto maior for a normal (N), menor deverá ser o
coeficiente de atrito (mi) e vice-versa. Isso é certo?  R

Qual é o mais baixo som que o ouvido humano é capaz de escutar, e a que ele se compara?  R

Eu gostaria de saber qual a diferença entre ondas IONIZANTES e NÃO IONIZANTES.  R

Qual é o significado físico da energia livre de Helmholtz?  R

Os Sabres-de-luz de Guerra nas Estrelas são constituidos por um feixe de luz continuo, o laser;
mas como eles detêm o feixe para estabelecerem o tamanho do sabre? Será que existe essa
possibilidade?  R

O que é velocidade de escape?  R

Por que as torres do World Trade Center ruiram minutos após terem sido atingidas pelos aviões
(lembrando um prédio implodido)?  R

O uso da água aquecida pelo sol causa algum problema de saúde como asma, gripe, infecção na
garganta, etc?  R

Trabalho como representante de varios frigorificos do pais aqui no Rio de Janeiro e tenho
constantemente problemas com diferenças de peso aferidas em balanças rodoviarias. A minha
duvida e se seria expressiva a oscilaçao de peso em funçao da diferença de altitude entre as
cidades de origem e destino da mercadoria. As carretas trazem aproximadamente 26 toneladas
de mercadoria e normalmente ocorrem faltas de peso de ate 300 kg.  R

Como é o funcionamento de uma bomba de vácuo que utiliza água para gerá-lo?  R

Ouvi algo sobre Partícula de densidade negativa, procurei em livros de 3ºgrau, mas não obtive
resposta. Poderá mesmo existir uma particula de densidade negativa?  R

Vou fazer uma cintilografia da tireóide. Injetarão na minha corrente sangüínea o isótopo
Tecnécio. O que pode acontecer comigo durante o exame?  R

No filme Apolo 13 o ator Tom Hanks está escrevendo com uma caneta esferográfica onde não
tem gravidade. É possível?  R

A cor verde das folhas tem algum significado físico?  R


Gostaria de saber por que se aquece a água para que ela limpe melhor.  R

O que é uma P-brana?  R

Que tipo de força a mente humana deveria produzir para mover objetos? Isto é possível?  R

Stephen Hawking, no livro *O universo em uma casca de noz* fala de universos paralelos em
branas diferentes, separadas por uma dimensão especial. Só a gravidade se propaga entre as
branas. Minhas perguntas são: 1) Se é tecnicamente possível construir um dispositivo análogo ao
que possuimos para geração e captação de ondas eletromagnéticas, com as ondas
gravitacionais? 2) Este possível *radio gravitacional* nos possibilitaria comunicarmos com o
mundo paralelo?  R

Quando o microondas é ligado para aquecer alimentos, as formiguinhas( do açucar) que por
ventura estão lá dentro não morrem, porque?  R

Qual a origem da unidade de comprimento Angstrom?  R

Por que há apenas 5 tipos de poliedros?  R

Gostaria de saber como faço para identificar num ímã qual lado é pólo norte e qual é o pólo
sul?  R

Por que quando desparafusamos algum aparelho eletrônico, os parafusos ficam


eletromagnetizados?  R

Como se explica o fato da bandeira americana tremular na lua, se não existe ar por lá?  R

Recentemente, o Daniel Doro postou um endereço de um arquivo PDF que falava sobre as Dp-
branas. Nesse arquivo, mostra-se uma equação do *Einstein Universe*, onde a energia Casimir
está presente. Gostaria de saber qual a importância de uma energia como a Casimir para as
teorias de membranas e o que é esse tal *Einstein Universe*. Grato!  R

Qual é o estado físico da singularidade de um buraco negro?  R

O Universo realmente caminha para uma morte térmica? Baseado na entropia e a segunda lei da
termodinâmica como posso responder a esta pergunta?  R

O que é wormholes? Existe? é perigoso?  R

O que é cosmologia quântica?  R

Duas recentes experiencias, a de Taleyearkhan (março-2002) e a Suslick-Didenko (julho-2002),


publicadas na Science e na Nature, confirmaram a fusao fria Mais informações estao disponiveis
no site http://ciencia2000.cjb.net. Gostaria que comentassem o assunto.  R

De acordo com a relatividade, um corpo em movimento tem sua massa aumentada, e quanto
mais rápido esse corpo estiver se movendo, maior será o aumento de sua massa. Mas encontrei
uma espécie de círculo vicioso com relação a isso. Se um corpo aumenta sua velocidade, sua
energia cinética aumenta. Com isso, sua massa aumenta (de acordo com E = mc^2). Mas se a
massa do corpo aumentar, sua energia cinética também irá aumentar, e assim sucessivamente.
Como se explica isso?  R

Como funciona o teletransporte?  R

Como funciona um holograma?  R

O que é um transdutor? Pra que serve? Como ele funciona?  R

O que foi o principio da complementaridade de Bohr?  R

Dois aviões partem de um mesmo ponto, à altura do Equador, para darem a volta ao mundo. Cada
avião leva consigo um relógio com as horas acertadas antes da viagem, e cada relógio confere
sua hora com o horário do local da partida. Os aviões partem e viajam com a mesma velocidade
igual e constante. O primeiro avião viaja em sentido direto: igual à rotação da Terra de oeste para
leste. O segundo, em sentido retrógrado, de leste para oeste. Sabemos que os dois chegarão
juntos porque o movimento da rotação da Terra e o avião "fazem" um sistema inercial (galileano).
Pergunta: quais os efeitos relativísticos em seus respectivos relógios; como eles ocorrem e por
quê.  R

Por que quando você está cortando uma cebola e está com um palito de dente na boca, o gas
que a cebola libera não o faz chorar?  R

No livro "Faster than Light, Superluminal Loopholes in Physics", Nick Herbert afirma que o
movimento super-luz não é vetado pela relatividade. Ele cita 14 situações em que seria possível
ultrapassar a velocidade da luz, como, por exemplo: quando a interseção das lâminas de uma
tesoura forem longas o suficiente e estas sejam fechadas em uma velocidade próxima de c; o
elétron que forma o ponto em um osciloscópio; um sinal radiofônico viajando através do plasma;
a expansão do universo instantes após o Big Bang; o material emitido por alguns quasares; etc.
Alguém pode comentar isso?  R

Se o electrão de um átomo colidir com o núcleo a sua posição estaria determinada. Poderia isso
acontecer?  R

Gostaria de saber se uma pessoa exposta ao sol, através de um vidro (p. ex. de um automóvel)
pode se queimar (ou bronzear) da mesma forma que alguém que toma sol ao ar livre. Os raios UV
atravessam qualquer tipo de vidro?  R

Li em uma revista, que uma nave pode usar a gravidade de um planeta para aumentar sua
aceleração durante uma viagem espavial. Como isso é possível?  R

Como funciona o display de cristal liquido?  R

Gostaria de saber como funciona a parada de segurança de um elevador caso ele tenha os cabos
de aço rompidos.  R

Que substâncias possuem os objetos que brilham no escuro?  R

Densidade optica é um termo que está sendo muito usado nas imagens produzidas em
informatica. Como podemos definir a densidade optica? O que é medir a densidade optica de
uma imagem? Gostaria de receber referencias bibliograficas. Agradeço a atençao.  R

Por que as ondas médias chegam mais longe à noite?  R

Gostaria de saber o que são raios cosmicos e de onde eles vem?  R

Gostaria de saber se alguém conhece uma poesia que fale sobre assuntos de física, caso
conheça, me informe que agradeceria imensamente.  R

Queria saber mais sobre os bósons e as quatro interações fundamentais.  R

Porque nas imagens que mostram o astronautas pisando na lua, não existem estrelas no céu (da
lua)?  R

Ainda é possível descobrir algum elemento químico novo?  R

Se enxergássemos em preto-e-branco saberíamos da existência das cores?  R

Por que as crateras são todas circulares e não oblíquas? Seja nos planetas sólidos, como nos
seus satélites, os impactos parecem ter sempre a direção normal à superfície. Veja o caso da
Lua, todas as crateras redondinhas.  R

Quando estou em um automóvel e por acidente um fio de alta tensão cai sobre o veículo, o que
devo fazer? Se eu sair do veículo levarei um choque?  R

Qual a diferença entre radar e sonar?  R

Por que as linhas do campo elétrico têm origem e fim e as do campo magnético não?  R
Gostaria de saber qual a explicação física para a questão de pessoas andarem sobre brasas e
não se queimarem.  R

Se o gás dos refrigerantes é CO2, e se colocarmos numa garrafa que já perdeu o gás um pouco
de gelo seco, o refrigerante se recuperaria? Tem algum risco para a saúde tomar este refrigerante
*reciclado*?  R

Por que os buracos negros não atraem os proprios gravitons, já que eles atraem os fotons?   R

O que vem a ser a hipótese de Curvatura de Weyl?   R

Não seria o Efeito Casimir uma forma de criar um motor perpetuo já que ele retira
indefinidamente energia do vacuo (nada)?   R

Li na Scientific American (ediçao brasileira) do mês passado sobre viagens no tempo e, falando
de buracos de minhoca, eles disseram ser previsto pela teoria estabilizar um buraco de minhoca
introduzindo energia negativa no sistema, usando, para isso, em espelho em movimento (que
originaria o Efeito Cashmir). Isso procede? Alguem poderia explicar direito isso?   R

Sobre os Campos de Higgs: sei que não devemos levar a sério as analogias tiradas da física
clássica para explicar física fundamental, mas uma coisa me intriga. Quando leio que a interação
das partículas elementares com o bóson de higgs dá-lhes inércia e massa e, portanto, tornam
difícil sua aceleração fica fácil de imaginar. Contudo, *massa\inércia* é também a dificultade de
parar algo em movimento retilíneo uniforme. Como a interação com os campos de higgs podem
dificultar uma desaceleração? A maioria dos autores de textos relacionados ao tema parece não
se preocupar em distinguir a interação com os higgs e o cotidiano atrito da física clássica.
Obrigado.   R

Sabendo que a matéria é feita por 90% de vazio, porque não me enterro quando bato com os pés
no chão?   R

O que é um Sóliton? Um tipo de onda que não se dispersa? Porque tem esse nome? Uma corda
da teoria das Supercordas seria um Sóliton? Agradeço.   R

Se a agua ferve a temperatura de 100 graus Celsius, como uma poça d`agua evapora se ela nao
está a 100 graus Celsius?   R

Se um nêutron (composto por 2 quarks para cima e 1 para baixo) decai para um próton (2 quarks
para baixo e 1 para cima) mais um elétron e um anti-neurino. Devo entender que quando o quark
para cima transforma-se num quark para baixo e ele emite um elétron e um anti-neutrino! Se
assim for posso considerar que o elétron e o anti-neutrino são de certa forma componentes
internos do quark para cima?   R

Vi uma vez num filme, que os arabes no deserto conseguem esfriar agua de uma garrafa
molhando um pano com a agua contida na mesma e em seguida envolvendo a garrafa no pano e
enterrando-a na areia; gostaria de saber se isso é possivel?   R

Estou com uma dúvida. Afinal de contas, a Força de Atrito depende ou não da área de contato?
Fiz uma experiência onde foi demonstrado que o coeficiente de atrito estático não se altera
quando alteramos a área de contato entre os materiais. Mas porque então temos que encher um
pneu de bicicleta para andarmos mais facilmente? Quanto mais cheio estiver o pneu, menor será
sua área de contato... e mais fácil será para andarmos. A força de atrito só depende do
coeficiente de atrito e a normal?   R

Como se explica o fenômeno de levitação de sapos?   R

Segundo a Teoria da Relatividade, a luz se movimenta apenas no espaço e não no tempo. Então
porque a luz emitida pelas estrelas demoram milhares de anos até chegar ao planeta Terra?   R

Por que existe um ângulo de diferença entre os Pólos terrestres e a orientação da bússola?
Quanto vale esse ângulo? Ele aumenta com o tempo? Por quê?   R
Existe alguma forma de se transformar som em energia elétrica? Seria fantástico imaginar uma
rede de *receptores* ao longo de avenidas e túneis transformando o som dos automóveis em
energia elétrica, além de diminuir a poluição sonora...   R

Gostaria de saber como funciona uma maquina fotografica KIRLIAN (aquelas máquinas que
fotografam a aura humana). Se alguém souber sobre o assunto e puder me mandar um esquema
de como se monta uma máquina dessas, serei muito grato.   R

O que faz do Brasil o campeão mundial em quedas de raios?   R

O que aconteceria se um astronauta tentasse usar uma bússola em Marte?   R

Como medir a massa do sol?   R

Sempre ouvi dizer que num dia de chuva uma bola lançada por um jogador de futebol ao bater no
solo ela não torna a subir mas por sua vez aumenta muito sua velocidade. Isto é verdade?   R

Ao arrastar meus pés com sapatos de solado em couro no carpete da sala climatizada onde
trabalho acredito que fico energizado. No momento em que eu toco em algo com a unha levo
choque. O que é impressionante é que chega a sair um pequeno raio azul de uns 2cm e ouve-se
um estouro. Fizemos um teste e eu vim arrastando os pés, depois aproximei minha unha de outra
pessoa e essa também levou choque!!! Como isso acontece, é normal?   R

Em uma pergunta que fiz a respeito da aniquilação que ocorre entre neutrino e anti-neutrino me
disseram que a anti-matéria tem energia negativa. Mas e quanto à energia que compõe os
fótons? É positiva ou negativa? Se for positiva ou negativa, como os fótons podem dar origem a
elétrons e pósitrons?   R

Por que uma folha de papel em seu estado normal ao balançamos de um lado para o outro emite
um som , mas se pegarmos esta mesma folha e amassamos formando uma bolinha de papel e
em seguida abrimos esta bolinha , e voltamos a balançar ela de um lado para o outro ela não tem
mais som? Existe uma explicação fisica para este fenômeno?   R

Porque a eletrizaçao por atrito nao é eficiente em dias úmidos?   R

No filme Impacto profundo, um meteoro caia na terra, só que não consegui entender uma coisa,
no filme era falado que quando o meteoro caisse ia levantar uma grande poeira que duraria
alguns meses, mas se esse meteoro caiu no mar como pode ter levantado tal poeira? Em um
caso desses, na realidade, em se tratando de cair um meteoro no mar, seria possivel levantar
poeira? Ou se trata só de ficção?   R

Se a água é condutora de eletricidade , porque não tomamos choque quando estamos tomando
banho , se a água circula em contato direto com a resistência do chuveiro?   R

Como evoluiram as calculadoras? Como eram as primeiras calculadoras? Como é que elas
funcionavam? Que tipo de pessoas as utilizavam? Quem inventou as calculadoras modernas e as
antigas?   R

Como funciona o chuveiro elétrico e como posso falar sobre seu funcionamento para crianças de
4º série do ensino fundametal? Por favor indique alguma experiencia para que os alunos possam
visualizar o funcionamento do chuveiro!   R

Como era o funcionamento das primeiras geladeiras se nem tinha energia eletrica naquela
epoca?   R

Sobre a luz não conseguir sair do efeito gravitacional do buraco negro: sei que ela não sai pois
sua velocidade não é suficiente para escapar de tal força, é o mesmo quando disparamos um
projétil em direção para fora da Terra, se ele tiver velocidade inicial suficiente ele escapará da
gravidade terrestre... ok. Duas questões: 1) Somente buracos negros conseguem produzir
gravidade suficiente para não deixar a luz sair? ou poderia existir um astro *normal* com tanta
massa que isto também ocorreria, digo *normal* pois sua massa ainda não colapsou, ou seja,
ainda existe energia neste astro capaz de manter sua gravidade *sob controle*. 2) Sobre a
radiação de Hawking, que é emitida pelos buracos negros: esta radiação teria que sair mais
rápido que a luz? Ou seria um tipo de radiação sem massa?   R

É verdade que a Terra está perdendo seu potencial magnético e que esta perda influencia nosso
organismo?   R

Ok, números reais + imaginários = complexos. Já ouvi falar de outros conjuntos numéricos que
não podem ser demonstrados num eixo cartesiano. Quais outros conjuntos numéricos existem?
Qual a função deles hoje em dia?   R

O que aconteceria se eu unisse uma antena externa a um prato parabolico e as ligasse ,com um
cabo de antena,numa televisão? Eu conseguiria o efeito esperado, ou seja, uma melhoria na
recepção da imagem?   R

Por que é que quando se atira uma pedra a um lago ....forma varias ondas e nao uma so?   R

Por que a Alquimia não prosperou? Com toda a tecnologia disponível, por que não podemos fazer
ouro a partir de outros materiais?   R

Qual é a relação, se é que existe, entre o potencial eléctrico e o potencial gravítico?   R

A constante de Plank é um numero irracional?   R

Como é considerado o VIDRO fisicamente? Liquido ou solido?   R

Sabemos que o aumento da pressão, seja por um choque mecânico por exemplo, desorientaria o
alinhamento dos átomos do imã fazendo com que este se desmagnetizasse por um determinado
instante, assim como o aumento da temperatura. Com base nisto, como eu posso bloquear a
atração magnética de aapenas uma região do imã sem aumentar a temperatura e sem usar
choques mecânicos de modo que esta parte se tornasse magneticamente isolada sem
comprometer a polaridade das demais regiões deste imã? Existe algum estudo quanto o
isolamento de ondas magnéticas?   R

Existem realmente fótons virtuais, são eles os portadores do eletromagnetismo? Os fótons reais
portadores da luz são também os portadores das ondas de rádio, raio X, ultravioleta,
infravermelho porém possuindo um comprimento de onda e frequência diferentes?   R

Li por aí que para se utilizar a velocidade de dobra seria necessário curvar o espaço-tempo. O que
seria necessário usar para se fazer isso? Energia negativa? Como um dia poderemos fazer isso,
se realmente for possível?   R

Porque razão nas viagens de regressso à terra do vaivém espacial, há cuidado com o angulo de
entrada na atmosfera. E o que é o angulo de entrada na atmosfera?   R

Já existe alguma forma de dissociar a água (eletrólise) e aproveitar o hidrogênio na combustão ?


O H seria combustível?   R

Pq a relatividade geral e a teoria quântica não se encaixam? O que os físicos estão fazendo para
unificar essas duas leis? Caso descobrissem uma equação única, poderíamos responder todas
as questões do universo?   R

Como pode um elétron estar em dois lugares ao mesmo tempo?   R

A idéia central é a conservação da energia. Tenho uma mola de aço. Comprimo-a e a coloco
(comprimida) em um recipiente contendo um ácido altamente corrosivo e que então irá corroer a
mola fazendo-a desaparecer. A pergunta é: Se a mola estava comprimida ela tinha energia
potencial armazenada. Se de fato a energia se conserva, para onde é que foi essa energia
potencial se a mola desapareceu?   R

O CO2 em estado sólido é chamado de gelo seco, e em estado gasoso de gás carbônico. Não
existe o estado líquido? Se sim, qual o seu nome. Se não, por que isso acontece (ele sublima de
uma vez, tipo a naftalina?)   R
Sobre a crise de energia elétrica, por que ninguém nunca teve a idéia de construir usinas *talasso-
elétricas* no nosso litoral, que aproveitassem as ondas do Oceano ou mesmo as correntes
marinhas para mover as turbinas, exatamente como as hidroelétricas aproveitam o fluxo dos
rios?   R

O elétron tem raio finito?   R

Se Vênus dá uma volta em torno do Sol mais rápido que a Terra, por que conseguimos avistar
Vênus o ano todo?   R

Que é espuma Quântica?   R

Se estivesse sentado em uma carga em movimento eu perceberia o campo magnético produzido


por ela?   R

Eu tenho um notebook e ele está estragado, mas o lcd, tela de cristal liquido está funcionando, eu
gostaria de saber se tem como funcionar ele como um monitor, e se tem como, gostaria de saber
como fazer isso.   R

Fisicamente falando, gostaria de entender o funcionamento do GPS (sistema de posicionamento


global).   R

As bombas nucleares podem perder seu efeito se não forem usadas logo, devido a desintegração
de seu material radioativo?   R

Em linhas gerais, qual o aparato tecnológico necessário para a construção de uma bomba
atômica? O Brasil dispõe de tal aparato? Em caso positivo, em qual(is) instituto(s) ou
lasboratório(s) possivelmente seria fabricada a bomba atômica brasileira?   R

Se oxigênio é gas e hidrogênio é gas, porque a água é liquida?   R

Ontem eu me encontrava com meus irmãos e cunhado formados um em geologia e o outro em


geografia e meu esposo em engenharia, houve uma breve discussão porque meu irmão e meu
cunhado afirmaram que o raio saíria da terra ou seja de baixo para cima e não saíria das nuvens
de cima para baixo. No momento não sabia quem tinha razão. E agora lhe pergunto o raio sai da
terra de baixo para cima ou cai das nuvens de cima para baixo?   R

Por quê o rendimento da transmissão da energia elétrica em corrente contínua é pior que em
corrente alternada?   R

O que é plasma de quarks? Há evidências teórico-experimentais de sua existência?   R

Considerando que a formula da água é H2O, o que ocorre na *oxigenação* da água nos
aquários?   R

Minha casa esta entre uma avenida e um grande muro, os quartos estão voltados para o muro. Eu
recebo o som como se estivesse na avenida. Tenho como diminuir este ruido com plantas que
venham absorver? Plantaria dos dois lados da casa, uma fileira na calçada da avenida e outra
emparelhada com o muro. Isto iria ter uma diminuição de ruido?   R

É possível ferver a água sem aquecê-la?   R

Quais os materiais (principalmente os de fácil obtenção no mercado) poderei usar como


eletrodos na eletrólise de solução aquosa de NaCl? Poderei utilizar os dois eletrodos de
*grafite*?   R

Se os vulcões são tão perigosos porque é que há tantas pessoas a viver perto deles?   R

O que é TEMPO?   R

Acabei de ler uma resposta sobre eletricidade estatica armazenada no corpo, adorei a dica da
moedinha e gostaria de saber se esses pequenos choques que levamos ao tocar outra pessoa ou
objetos metalicos pode prejudicar o embriao ou feto. Esses choques estao acontecendo comigo
ja faz um tempo, como perdi um bebe ha alguns meses fico preocupada que essa pode ser uma
das causas.   R

Quando estamos viajando em rodovia a noite e avistamos uma cidade, as luzes das mesmas
ficam piscando. Eu queria saber realmente o porque delas piscarem quando estamos
quilômetros distantes delas e quando estamos perto elas não piscam... Não sei se já repararam,
mas quando estamos distantes e tem alguma luz acesa perto de nós, esse efeito do piscar das
luzes distantes não é percebido.   R

Sou professor e por mais de uma vez, ao dar aulas de termologia, alunos me dizem que, quando
estão nadando e começa a chover, têm a sensação de que a temperatura da água dentro da
piscina fica mais alta do que estava. Esse fenômeno realmente ocorre ou seria apenas uma falsa
sensação do corpo da pessoa?   R

Por que sai vapor do chuveiro se a água não chega a 100°?   R

Qual foi duvida ou erro que encontrou Einstein nas Leis de Maxwell que o motivou a desenvolver
sua teoria da relatividade?   R

O que é a Teoria do Caos? Pode falar um pouco a respeito?   R

Como funciona a lata de cerveja com a serpentina que quando aberta se resfria sozinha?   R

Eu queria saber como se faz para medir a potencia do motor de um carro com um dinamo... em
revistas decarros tem *(...)360 cavalos a 6200 rpm aferidos com dínamo(...)* e eu gostaria de
saber como é o dinamo usado por esses caras.   R

Por que o corpo humano induz um aumento de temperatura (febre) quando é atacado por um
organismo estranho?   R

Existe a possibilidade de ocorrer efeito fotoelétrico em superfícies não-metálicas?   R

MEU PC NÃO TEM O FIO TERRA...LEVEI CHOQUES NELE JÁ ! MORO EM PRÉDIO......E VEIO UM
ELETRICISTA AQUI E FALOU QUE É MUITO DIFÍCIL FAZER ATERRAMENTO AQUI, PELO FATO DE
SER PRÉDIO ! COMO FAÇO PARA PARAR DE LEVAR CHOQUES? TEM UMA OUTRA MANEIRA SEM
SER ESTA DO FIO TERRA?   R

pq o pneu mais fino, por exempolo de uma bicicleta speed, tem que ser enchido com maior
pressao que o pneu de um carro?   R

Alguns físicos afirmam que a força de atrito não realiza trabalho. Essa afirmação está correta?   R

Quero fazer algumas experiências com tubo de Crookes e pretendo fabricar o meu. Existe aqui no
Brasil algum curso (mesmo em universidades), livro ou site onde ensinem a fabricar os tubos de
vidro. Ouvi dizer que este curso chama-se hialotecnia.   R

De acordo com a termodinamica, por que é mais perigoso uma queimadura provocada por vapor
de agua a 100 graus do que por agua liquida na mesma temperatura?   R

Quando uma garrafa de cerveja está em um freezer, com uma temperatura de -5ºC, no estado
líquido e a pegamos pelo meio, porque o líquido se congela, se estamos lhe fornecendo calor?   R

Como se obtem o gás hélio?   R

Posso dizer que o tempo é curvo? Há algo para provar isso, como por exemplo, uma equação?   R

O Gás Ozônio utilizado na esterelização (higienização) do ar condicionado para automóveis


causa algum dano a saúde de quem o aplica nas lojas especializadas ou do proprietário do
veículo?   R

Afinal, o neutrino sofre interação com o campo magnético solar ou não? Será que é dai que ele se
transforma em neutrino do muon e do tau? E para onde vão os neutrinos não detectáveis?   R
Gostaria de saber qual o processo utilizado para medir altas temperaturas (15.000 ºC, 100.000
ºC, etc),como por exemplo: do sol, do epicentro da explosão de uma bomba atômica, ou seja,
temperaturas que pela lógica derreteriam qualquer tipo de material que pudesse ser empregado
na fabricação de um termômetro.   R

Quando transpiramos, ganhamos ou perdemos Entropia?   R

Se a hora é 1/24 do dia, por que a rotação da Terra é de 23h, 56min e 4 seg. E a diferença, quando
e como é acertada?   R

O que causa a microfonia (aquele som agudo devido a aproximação de um microfone a um alto-
falante do mesmo circuito do microfone)?   R

Estou fazendo uma pesquisa para medir o tempo de vida util de uma pilha, qual seria o melhor
método para medição? ( no físico ) Sabe-se quando a pilha estiver em paralelo com resistor de
valor igual a sua resistência interna sua potencia fornecida será maxima, experimentalmente é
verídico? Me disseram que, para medir o tempo de vida de uma pilha comum, duracell, podemos
medir com um resistor de 75 ohms diz a ABNT, confere?   R

Porque a atmosfera apresenta sobre os pólos menor espessura que sobre as regiões
equatoriais?   R

Como podemos criar um campo gravitacional em estações espaciais ou dentro de naves que se
encontram no espaço?   R

Por que temos a impressão de ouvir o barulho do mar quando encostamos o ouvido numa
concha?   R

Por que as órbitas dos planetas, ou de qualquer objeto submetido a uma força resultante central
que varia com o inverso do quadrado da distância, é uma cônica?   R

O que é matéria escura?   R

Como, a partir da observação dos choques das partículas em um acelerador de partículas é


possível detectar a presença de novas partículas?   R

Qual é a galáxia mais proxima da nossa?   R

O Daniel falou em uma pergunta que a antigravidade só existe se existir energia negativa. Mas
considerando que no eletromagnetismo os fenômenos de atração e repulsão de cargas elétricas
são perfeitamente explicáveis sem essa exigência, por que existe essa complicação para a
gravidade?   R

Imagine que pudéssemos ir exatamente até o centro gravitacional da Terra e abrir um buraco
perfeitamente esférico lá, de mais ou menos um metro de raio. Então, no centro da esfera, nós
posicionamos uma pedra perfeitamente esférica e soltamos. O que acontecerá com a pedra? Ela
cairá para algum lado ou ficará estática, *flutuando*?   R

Na minha cidade as televisoes nao conseguem obter um bom sinal, por isso na maioria das
casas há uma antena parabolica. Coloquei uma antena normal com um buster mas mesmo
assim nao consigo obter um bom sinal. O que poderia fazer para conseguir um bom sinal sem
precisar comprar uma antena parabolica. Há alguma alternativa de baixo custo para conseguir
assitir a TV com uma boa qualidade de imagem?   R

O que é memória spintrônica?   R

Como foi calculada a massa da Terra?   R

Porque os planetas estão aproximadamente no mesmo plano e principalmente porque movem-se


para o mesmo lado?   R

Como funcionam os cartões magneticos e porque é tão facil clona-los?   R

Como se dá o processo de geração de energia elétrica em uma usina hidroelétrica?   R


Como calcular a declinação magnética da cidade de São Paulo? Por que esse número é alterado
anualmente?   R

O que é o CONDENSADO DE BOSE/EINSTEIN?   R

Como Isaac Newton deduziu a fórmula |F| = GMm/R² da Lei da Gravitação Universal?   R

É correto colocar roupa para secar de trás da geladeira?   R

Tenho um equipamento de som de 500watts RMS ou seja real; Em ambiente aberto (praça)
quantas pessoas eu posso atingir; e também em ambiente fechado os mesmos 500 watts
quantos eu posso atingir?   R

Por que levo choque ao sair do carro? Vocês têm uma solução. Hoje, por exemplo, levei tanto
choque que estou até com dor de cabeça. Já experimentei vários tipos de solas de sapato e não
adiantou. Levo choque ao sair do carro e colocar a chave para fechá-lo. Chega a sair faisca. Se
não faço este procedimento, o choque vem quando coloco a mão para a abrir o portão etc. Enfim,
é um desespero. Outro dia, ao abraçar uma pessoa, levei um superchoque. A pessoa até
percebeu.   R

Qual foi a causa provável do mar vermelho ter aberto para o povo de israel passar?   R

Se as Leis de Newton só são aplicadas 'corretamente' em referenciais inerciais(v = cte), porque


existe a 2º Lei de Newton que envolve a aceleração?   R

A energia dos raios pode ser aproveitada?   R

Qual é o mais antigo continente do planeta? Não é em relação ao mais antigo a ser habitado
pelos povos, mas sim ao que surgiu primeiro.   R

Já li em vários lugares o que é um transistor, mas ainda não entendi. Poderia alguém me
dizer?   R

De onde veio a água de toda a Terra?   R

Gostaria de saber se é possivel transformar corrente contínua em alternada. Se for gostaria de


saber como e que tipo de aparelho é usado.   R

Gostaria que me informassem sobre o que é exatamente um elemento Peltier, tem pra comprar?
Onde? Quanto custa?   R

Qual o raio de distância, que é necessário para a vida na terra sobreviver, a uma explosão de
estrela supernova como ETA CARINA?   R

Todos dizem que é necessário a instalação do fio terra, disso eu já sei. Só que ainda não
encontrei um bom eletricista que me dissesse como é feito a instalação do fio terra das
lavadoras de roupas que operam com água fria,que é aquele fio verde que vem externamente.
Quando se trata de uma lavadora que opera com água quente, o fio terra já vem embutido no plug
tripolar, assim, a instalação é só uma, toda vez que ligarmos o plug na tomada, estaremos ligando
também o fio terra, assim como toda vez que desligarmos, estaremos desligando tudo também.
O que eu quero saber como é feito a instalação do fio terra dos equipamentos que tem plug
bipolar e o terra é externo. Como posso fazer essa instalação de forma que possa mover o
equipamento sempre que quiser, apenas desconectando o fio terra, como se fosse um plug na
tomada?   R

Por que o dióxido de carbono permite a entrada da radiação infravermelha vinda do Sol, e não
permite a sua saída, ocasionando o chamado efeito estufa?   R

Fiquei sabendo que existe uma diferença de altura entre os dois oceanos (Pacífico e Atlântico)e
que é esse diferencial que faz com que o Canal do Panamá funcione. O que aconteceria se
destruíssemos as duas comportas em cada extremo do Canal? E a teoria dos vasos
comunicantes? Toda a água do oceano *mais alto* escoaria para o oceano *mais baixo* e
teríamos uma catástrofe mundial, com alteração de nível dos aceanos? Estou desesperado de
curiosidade!   R

Uma certa lenda de mesa de bar afirma que beber cerveja com canudo embriaga a pessoa mais
rápido do que beber sem o canudo. Com canudo, ingere-se o líquido que está no fundo do copo.
Sem canudo, ingere-se o líquido da superfície. Existe uma diferença na concentração de álcool
entre o fundo e a superfície do copo de cerveja para que a lenda se comprove?   R

A ausência de gravidade provoca alguma alteração na digestão dos astronautas?? Qual é o


mecanismo usado por eles pra impedir o refluxo dos alimentos?   R

Os cientistas ainda não conseguiram atingir o zero absoluto. Mas já chegaram perto: apenas 170
bilionésimos de grau acima do zero (Esses dados são de dez. 1996, retirados da revista
superinteressante). Gostaria de saber quanto os cientistas conseguiram atingir, hoje (em relação
ao zero absoluto)?   R

Temos um microondas Samsung 110v. Minha esposa ligou ele na fonte de 220v, e queimou a
fonte. Pergunto a vocês se todo o microondas foi queimado, ou se apenas a fonte de energia?
Posso transformar ele para 220v?   R

O que é exatamente o PRINCÍPIO DA INCERTEZA?   R

Sei que basicamente tudo é formado por átomos, mas o que existe entre o núcleo e a eletrosfera
de um átomo?   R

Sabemos que por convecção os gases mais quentes sobem e os mais frios descem, então
porque quanto mais alto, mais frio?   R

As teorias de Fran de Aquino sobre a antigravidade tem algum fundamento científico? Se tem,
porque quase ninguém fala acerca dela?   R

Minha empresa está entrando no ramo de iluminação-ambiente p/ festas... ou seja, salões,


buffets, espaços em geral... com duração de evento de até 06 hs continuamente. Pergunto: Qual
a bitola exata de fios pp, ou outro qualquer, cuja wattagem possa chegar a 25.000? Existe uma
forma aritmética para eu saber exatamente qual bitola usar? exemplo: sendo (x)wattagem
(y)fio.... ou algo parecido? Posso usar benjamins de 15a p/ três ligações de lâmpadas de 1000
watts? ou seria melhor aqueles tabletes ou circuito de linha? O que signifiva KVA? Se for usar
gerador no evento, qual a mínima capacidade em KVA´s que a unidade precisaria ter para
abastecer até 25.000 wattts?   R

Gostaria de saber como faço para saber se as pilhas (1.5V) que tenho em casa ainda podem ser
usadas ou se podem ser descartadas. E no caso das pilhas (tipo moeda) para relógios e
calculadoras? E baterias de celular? Como usar o multimetro para saber isso?   R

Em pintura as 3 cores basicas são o vermelho, o amarelo e o azul. Se eu dividir um circulo em 3


partes iguais, e pinta-las destas cores, fixar o centro e girar o circulo tudo fica branco. Num
arcoiris as cores tambem estão nesta ordem. Por que num televisor as tres cores basicas são o
vermelho, o azul e o verde?   R

Quando movemos um membro do nosso corpo alteramos o nosso centro de massa. Como isso é
possível se apenas uma força resultante externa pode mover o centro de massa?   R

Nas naves espaciais o que é feito com o CO2 produzido na respiração dos astronautas?
Podemos retirar o carbono e deixar o oxigênio e de que maneira econômica fazemos isto?   R

Como um motor híbrido, que usa gasolina e álcool simultaneamente, consegue funcionar? Que
tipo de sensor ele usa?   R

Um carro gasta menos combustível em maiores altitudes?   R

Vi há tempos atrás um *arco íris* às 21:00 horas. Tratava-se de um *arco* semelhante ao arco
íris, porém não havia cores a não ser a preta que se destacava da cor do céu naquele momento.
Como se forma este fenomeno?   R
Quando a agua entra em ebulição, ela sofre alguma alteração na sua composição? Altera o seu
sabor? Porque varios produtos solicitam que se use a agua quando ela inicia a fervura?   R

Por que podemos ouvir sons em dias de umidos que nao podemos em dias secos?   R

Como construir um artefato qualquer para gerar vácuo?   R

Gostaria de saber porque as imagens são verdes (na maioria das vezes) quando feitas com
filmadoras que utilizam o infra-vermelho para filmagens noturnas.   R

Porque quando há uma fina camada de água sobre o azulejo e encostamos o dedo ensaboado
nela parece que a água é expulsa de onde o dedo tocou e depois continua expandindo pro lado
essa área?   R

Como posso diminuir a umidade de 75 para 60% de uma sala com o uso de ar condicionado ou
calefação?   R

O que é cavitação e por que ela ocorre?   R

Queria saber mais acerca desta teoria «Tempo e mecânica quântica clássica: indeterminação
contra a descontinuidade». Um estudo publicado na revista científica Foundations of Physics
Letters, desenvolvido por Peter Lynds, um neozelandês de 27 anos.   R

Sou professora em uma regiao muito carente de recursos. Tenho um aluno de 10 anos ( nao
alfabetizado). Percebi que ele enxerga letras de cabeça para baixo e lê da direita para a esquerda
! Isso e possivel?!!   R

Por que as células solares são tão caras? Ou melhor, quais são as dificuldades encontradas na
sua construção?   R

O que foi a CATÁSTROFE NO ULTRAVIOLETA?   R

O que é a Teoria do Mar de elétrons?   R

Porque as primeiras pedras de granizo são menores e as últimas maiores?   R

Por que escutamos barulho advindos de fios de alta tensão?   R

Qual a origem do nome buraco negro?   R

A estréia do filme "O exterminador do futuro III" criou uma discussão entre eu e um amigo. No
primeiro filme da série, é dito que o personagem John Connor _líder dos humanos na guerra
contra as máquinas no futuro_ é filho de um homem que veio do futuro para defender a mãe dele,
Sara. Só que esse homem, na verdade, nasceu depois do John Connor e foi criado por ele quando
criança. Meu amigo diz que isso é um "erro" do roteiro e que deveria haver um "primeiro pai" do
John Connor, nascido antes dele. Eu respondi que, em termos abstratos, isso é possível, pois o
passado, o presente e o futuro coexistem e o filme justamente brinca com esse nó temporal. Mas
ele não se convenceu e eu gostaria que vocês nos ajudassem nessa questão.   R

>É possível a existência de seres viventes em um universo paralelo? Poderíamos estar sendo
observados por eles?   R

É verdade a afirmaçao que o proton a distancias muito proximas de outro proton passa a ser
atraido ou seja se colam?   R

O que vocês acham da declaração de Cesar Lattes, dizendo que Einsten atrasou a Ciência em
100 anos e que as suas fórmulas já teriam sido feitas por Poincaré?   R

Se admitirmos que a entropia do Universo tende a um máximo, neste máximo todos os pontos do
universo teriam 'informação' zero. Ou seja, não haveria diferença entre pontos no Universo, e o
próprio conceito de 'distância entre pontos' não seria aplicável. Ora, se não podemos diferenciar
um ponto do outro, todos os pontos são (matematicamente) congruentes. Daí, podemos chegar a
um 'Big Crunch' sem ter que passar por uma fase de colapso do Universo. Meu raciocínio faz
sentido?   R
Como as leis da mecânica quântica podem ser aplicadas em àreas tão complexas como o direito,
uma vez que o direito também se trata de sistema complexo?   R

Ao longo de nossas vidas nos deparamos com exclamações do tipo como o ano voou ou voce já
teve a sensaçao de que o tempo esta passando mais rápido. Essas sensações, aparentemente
inócuas, não poderiam ser uma forma sutil de a natureza dizer à nossa intuição que, é verdade, o
tecido do espaço tempo está sim expandindo-se a taxas crescentes - acelerando - e que nós
estamos sentindo os seus efeitos?   R

Gostaria saber porque é que sonhamos? Gostaria de compreender esse processo, o processo do
sonho.   R

Como fica a Teoria M. se descobriram que a Teoria do BigBang estava furada? Já que de acordo
com os cientistas que descobriram um planeta fora do nosso sistema solar, que tal planeta é
milhões de anos mais velho que a explosão do BigBang.   R

Veja bem, no paradoxo criado na hipótese de viagem no tempo através de uma ponte de Einstein
-Rosen onde o indivíduo volta ao passado e mata a própria mãe antes dele mesmo nascer, não se
poderia discutir a possibilidade baseando-se na teoria dos universos paralelos? Nesse caso eu
não estaria criando (ou visitando)um novo universo de possibilidade, onde eu não existiria como
'eu'?   R

Ao mergulharmos parcialmente um giz azul em um copo com água e o retirarmos, notamos que a
parte úmida adquire uma tonalidade azul mais escura. Físicamente, qual a explicação desse fato
se a água é incolor? Tal fenômeno também é observado em pessoas louras, nas quais os cabelos
escurecem ao serem molhados. Qual a explicação física para isso?   R

É ilusão pensarmos que os alimentos quando frescos têm um sabor diferente daqueles que já
estão em algum tempo na geladeira? O que se tenta evitar quando colocamos alimentos na
geladeira?   R

Por que não é possível estacionar um carro num espaço um pouco maior que o seu comprimento
(fazer a baliza) colocando-se primeiro a sua frente? Temos sempre que estacionar em marcha
ré?   R

Se os elétrons possuem massa, e a corrente elétrica e a passagem de elétrons por um condutor.


Então podemos considerar que flui matéria (massa) de uma usina hidrelétrica para um
residência?   R

Se o dia tem 23h e 56min, então, vejamos: Nossos relógios atrasam 4min por dia e 40 minutos a
cada Dez dias. Mas, então, por que em dez dias não percebemos o dia ficar mais claro ou mais
escuro, já que o dia devia escurecer 40 minutos antes?   R

Tenho 40 anos e estou tendo problemas com choque, pois qualquer metal ou pessoa que toco
levo um choque que não é fraco, gostaria de saber se vcs tem ideia do que posso fazer para isso
parar.   R

qual produto quimico eu posso utilizar para remover tinta de caneta esferografica?ja utilizei de
tudo um pouco e nao adiantou! ja utilizei alcool, benzina, eter , acetona, cloro , etc....... nada tira
essa tinta?   R

As distâncias percorridas pelos automóveis até cessar o movimento são variadas. Porque os
extremos de massa de comportam da mesma maneira? Isto é, tanto uma moto como um
caminhão carregado demoram mais para parar que um carro de passeio. Tem alguma coisa a ver
com sistema de freio, área de contato dos pneus com o pavimento?   R

Gostaria de saber porque sai um líquido da pilha e que líquido é esse.   R

O que causou a inclinação do eixo de rotação terrestre?   R

Que luz é importante ter num ambiente onde se exige uma boa visualização de cores, por
exemplo num papel?   R
É possivel comunicar com outras pessoas mentalmente? se sim como posso fazer isso? quais
os passos para fazer telepatia e comunicar com outras pessoas usando a mente?   R

O que significa a frequencia informada nos microprocessadores?   R

Gostaria de saber se as lâmpadas fluorescentes compactas que emitem luz branca podem ser
prejudiciais à saúde. Sinto um grande incômodo nos olhos quando estou em um ambiente com
este tipo de iluminação. Além disso, este tipo de lâmpada emite alguma radiação prejudicial?   R

Na eletrolise da água, porque o volume ocupado pelo hidrogenio é o dobro do ocupado pelo
oxigenio sendo que o átomo de oxigenio é bem maior que o do hidrogenio e o espaço é forçado
pela pressão da água. Duas moléculas de gás hidrogenio e uma de gás oxigenio ocupam o
mesmo espaço?   R

Qual a hipótese mais aceita para explicar o campo magnético da terra?   R

É uma dúvida muito simples e não tem muito a ver com física e sim geologia: qual será a
configuração continental da terra daqui a milhões de anos, já que no passado os continentes
estavam em posições diferentes?   R

Como é o funcionamento do microscópio de tunelamento? E do microscópio de força


atômica?   R

Pq. a chama da vela é voltada para cima, mesmo que a gente vire a vela?   R

Se alguem descobrisse a fusão nuclear fria ou inventasse um motor movido a agua a baixo custo
ou melhor ainda inventasse uma forma de obter energia abundante e barata. Como ele deveria
agir para que o mesmo não desaparece do mapa. Visto que tal descoberta provocaria interesses
gigantescos. Resumindo eu poderia realmente descobrir qualquer coisa deste tipo?   R

O que é água polar?   R

Existe um equipamento para suprimir o som? Motivo: Pessoal mal educadas colocam seus
carros proximos a minha janela e não convem ficar brigando com todo mundo, se um aparelho
pudesse suprimir o som, só bastaria ligar e ficariamos em paz.   R

Por que utilizamos uma luz vermelha numa sala de revelaçao?   R

Como funciona o relógio atômico?   R

O que é a Energia do Ponto Zero?   R

Existe algum tipo de pesquisa buscando criar *chips* de computador com componentes
biológicos, material vivo no lugar de sílica...que relação há entre construção de novos
componentes e *nanotecnologia*?   R

Já que dizem que a pilha pode ser carregada no congelador, quais são os seus elementos
químicos e o que eles podem fazer ao corpo humano?   R

Baseado na má condutividade elétrica da água destilada é correto afirmar que se um corpo


humano estiver num recipiente, imerso em água destilada, e nós introduzirmos as duas pontas de
um fio ligado a uma tomada neste mesmo recipiente o corpo imerso não levará um choque?   R

Gostaria que alguém escrevesse algo sobre o *Projeto Filadélfia*.   R

Possuindo o nosso universo 11 dimensões, (3 dimensões espaciais + 1 dimensão temporal + 7


dimensões adicionais) o que seriam exatamente estas sete dimensões restantes?   R

Existe alguma explicação física para o Deja Vu?   R

O que é navalha de Occan?   R

Desejo saber se é possivel criar um som que espante muriçocas, me falaram que existe um
programa que emite este som e é som de alta frequência.   R
O QUE É O PARADOXO EPR?   R

O que é uma bola-relâmpago?   R

Gostaria de saber se existe algum estudo pratico recente ou teoria sobre como controlar o campo
gravitacional para aplicaçoes, por exemplo, em aeronaves.   R

Por que o olho humano não é capaz de visualizar a onda emitida por LED de controle remoto mas,
quando olhamos esta onda através de uma camera digital a onda pode ser visualizada?   R

Gostaria de saber o que são neutrinos, e o que seria a "partícula de Deus" popularmente
conhecida.   R

Uma vez estava lendo um livro de ficção(Operação Cavalo de Tróia,J.J.Benítez)e nele aparece um
comentário sobre partículas chamadas swivels, que seriam a base para todas as outras
particulas.Elas existem na realidade ou são só fruto da imaginação do escritor?   R

O que ocorre com a resistência vascular pulmonar quando a pressão venosa e arterial pulmonar
aumenta?   R

Gostaria de saber sobre a absorção da radiação infra-vermelha pelos materiais opacos: ao incidir
a luz solar em uma superfície, boa parte da radiação IV é absorvida e é refletida a parcela visível
da radiação, ou seja, luz visível e também a parte de ultra-violeta. Gostaria de saber quanto do IV
é absorvido, se depende do material e cores, onde posso pesquisar sobre o assunto. Outra
pergunta é sobre o andamento das pesquisas sobre vidros em sanduíche com gel eletrocrômico,
que submetido a uma tensão (parece que 12V) absorveria IV, pesquisa que fiquei sabendo num
congresso aí em Fortaleza em 1999, que estava sendo pesquisado na USP, em São Carlos. Sou
arquiteto, M.Sc. em Conforto - iluminação e quero me aprofundar sobre o assunto. Agradeço a
atenção e convido a uma visita - www.sig-arquitetura.hpg.com.br.

É possivel se fabricar celulas foto-voltaicas com recursos caseiros, e se é, como posso fazer?   R

Eu quero saber a respeito da "radiação" de hawking. Segundo o seu livro *O Universo numa casca
de noz* , ele fala que um buraco negro emite radiação e que pode vir deixar de ter massa dentro
de um intervalo de tempo muito, muito longo. A pergunta é: Como isso é possivel, se nem a luz
consegue escapar de lá?? E mais, as tais particulas virtuais; a que foi sugada nunca vai sair de
dentro do buraco negro! Como ele fala que a massa do buraco negro vai acabar?? E a que
*escapou* não estava no buraco negro, mas na borda do seu horizonte de eventos. Por favor,
deem um fim nesse meu emaranhado de dúvidas.   R

Se voce usar um pneu de maior diametro no seu carro, obviamente que o carro irá andar mais
rápido para a mesma rotação de motor de quando usando o pneu normal. Se usando um pneu
um pouquinho maior, digamos 5% maior, isso não significaria que o motor, caixa de cambio, etc,
teriam rodado menos para completar os mesmos 100 mil quilômetros percorridos? e
consequentemente sofrido menores desgastes? além de que após andar os 100 mil quilômetros,
o odômetro do carro estaria marcando só 90 mil...   R

Matematicamente é sabido que se esticassemos um anel de aço ao redor do planeta Terra,


grudadinho no chão, tal anel teria um diametro de aproximadamente 12.760.000 metros de
diametro, o que daria aproximadamente 40.085.540 metros de circunferência. Agora, se
abrissemos esse anel e inserissemos só um metro a mais, agora com uma circunferência de
40.085.541 metros, obviamente o diâmetro do anel aumentaria para 12.760.000,32 metros de
diametro. Isso significa um aumento de raio de 16 centímetros. Tambem significa que se o anel
pudesse flutuar, se afastaria do planeta 16 centímetros em TODOS os pontos. Pergunta: Por que
mesmo comprovando isso matematicamente, temos a sensação de que 16 centímetros de altura
seria demais para um mísero metro de aumento em tal magnitude de diametro?   R

Já que não importa a velocidade de deslocamento de um veículo, a parte de baixo do pneu


sempre estará em velocidade zero com relação ao solo, isso significaria dizer que se um carro
em movimento passar com o pneu sobre (desculpe, não achei exemplo melhor) a mão de
alguem, não será a velocidade do carro, mas sim o tempo que o pneu ficar esmagando a mão da
vítima, que causará maior estrago? Significa que quanto mais rápido o veículo passar,
possivelmente causará menor danos por estar pressionando a mão por menor tempo?
Significaria dizer que se o veículo passar a 1000 km/h, o tempo de permanência sobre a mão
seria tão pequeno que potencialmente não causaria dano algum? (1000 km/h significa cobrir 10
cm - tamanho da mão - em só 360 microsegundos) A mão humana não suportaria 300 a 400kg
(peso de um automóvel dividido por 4 pneus), durante a fraçãozinha de tempo de 0.00036
segundo?   R

Estou fazendo uma pesquisa sobre erros cinematográficos relacionados a fisica e gostaria saber
dos erros que vocês também conhecem.   R

O que vem a ser a SETA DO TEMPO, e qual a sua relação com a 2º Lei da Termodinamica?   R

Alguém pode me explicar, com clareza, (uma vez que ainda curso a 8° série) o que é um Quark?
De que forma os seres humanos tiraram essa idéia uma vez que nós ainda não podemos
enxergar os elétrons, prótons e nêutrons?   R

Ha alguns meses atras, a Inglaterra enviou uma sonda para Marte, no jornal onde li a noticia,
havia um infografico explicando o percurso que ela faria. O percurso seria helicoidal, ou seja, ela
orbitaria a Terra em orbitas cada vez maiores ate alcancar a orbita de Marte. Pergunto: Porque
este percurso tao grande? Nao seria melhor uma orbita direta?   R

Se a matéria escura é tão vital, determina o futuro do universoe a velocidade de rotação das
galáxias, porque não afeta as órbitas dos planetas?   R

A idéia que os neutrinos "direitos" podem ter massa elevadíssimas por instantes de tempo muito
pequeno foi confirmada? Sobre tais condições esses neutrinos poderiam ser a matéria escura?

Geodésica fechada no tempo? Caramba! Essa fechou foi o meu cérebro. Dá prá explicar?   R

O que é um strangelet? É verdade que um deles poderia destruir todo o Universo?   R

Quais são as influências que a Lua exerce sobre a Terra e sobre os seres vivos que nela
habitam?   R

Olá! Sou estudante de Engª de Materiais e em uma aula de química geral surgiu a seguinte
dúvida: Sabe-se que as moléculas de água se ligam por pontes de hidrogênio, em estado sólido
essas moléculas encontram-se mais próximas umas das outras, em estado líquido mais
afastadas, entretanto, porque o volume da água em estado sólido é maior que em estado
líquido?   R

Se houvesse um purificador de agua a base de *ozônio* para se purificar a agua em escala


industrial, para 25.000 e 250.000 litros, este poderia ser colocado em caixas de agua, piscinas, e
também para o tratamento da agua em fabricas? Outra pergunta : Qual seria o tempo necessário
para o consumo da mesma após ozonização? Existe algum tipo de lei que proiba o uso de agua
ozonizada nestes lugares citados? Uma vez que as industrias que fabricam produtos como
*Cloro* são fortes no mercado e não tem interesse que este aparelho seja fabricado, poderiam
ter ajustado alguma lei para o bloqueio da utilização do *ozonio*?

Por que se fala que vai acabar a água se ocorre o ciclo evaporação, precipitação, etc? A água que
evapora do mar não "chove" no continente alimentado as bacias hidrográficas, ou a evaporação
das bacias é que vai para o mar? É certo que há transformação de água (reações químicas de
baterias, misturas com cimento, etc) mas a água utilizado pelos seres vivos não é praticamente
devolvida para a natureza?   R

Estou em Portugal e estou a pensar levar para SC - Brasil, alguns electrodomésticos , que aqui
tenho, como por exemplo: arca frigorífica (220V 50HZ), máquina de café expresso (220V 50 HZ),
liquidificador (220V 50/60HZ), microondas (220V 50HZ), batedeira (220V 50HZ) ... etc ....
Qualquer um deles funciona aí, ou terei que fazer algumas alterações e quais? Desde já agradeço
a vossa ajuda.   R

No filme "O Segredo do Abismo", para desarmar uma bomba atômica, um mergulhador desce a
região abissal do mar a 80km de profundidade usando uma roupa de mergulho especial que lhe
permite respirar um "Líquido" para resistir a grande pressão, pergunto se seria possível ou se já
existe, ou é pura obra de ficção esta técnica?   R
Como funcionavam as antigas geladeiras a querosene. Por que não existem mais, eram
econômicas e confiáveis, já que não tinham peças móveis e nem faziam barulho?   R

Tenho um refrigerador que foi removido de um avião Boeing 747 convertido para cargueiro, que
funciona em 125 Volts e 400 Hz. Posso ligar esse aparelho na rede elétrica de 60 Hz?   R

É obrigatório, ou aconselhável zerar o termômetro antes de colocá-lo no que quero medir a


temperatura, ou é indiferente, o resultado não se alterará?   R

Foi-me dito que existe, fora do nosso espaço tempo, uma entidade apelidada de Mar de Dirac.
Seria uma sopa de energia que não corresponde a nada do que supõe o senso comum. Uma
'bolha' (ou defeito topológico) desprendeu-se desse Mar e deu origem ao nosso Universo.
Pergunto: 1-) Assimilei corretamente o conceito ? 2-) É possível que novas 'bolhas' se
desprendam dando origem a novos Universos?   R

Afinal de contas, o que realmente influi nas correntes de vento? Em se falando do litoral, eu ouço
falar muito em diferença de pressão, mas já vi, em Parnaíba-Pi, o vento soprar com velocidade de
60km/h das 6 até ás 8 da manhã; às 14h, estava 35km/h. Será que na parte da tarde não há mais
diferença de pressão? a maré, a lua influem realmente? Costumava ouvir na escola que de dia o
vento sopra do mar para a terra, e de noite da terra para o mar; Conheço bem o litoral do Pi, Ce, do
Ma, ES, Ba, Pb, e só vejo tamanha diferença de direção de vento qdo tem frente fria; qdo é época
de estiagem, eu costumo sentir o vento soprar, por dias, na mesma direçao. ( Sou velejador e
curioso, o lugar mais interessante que vi, foi Camocim, o vento sopra bem fraquinho até às 13h,
depois, a gente ver ele chegando, e é um Deus nos acuda para quem só tem vela grande).   R

Ao ter breve contato com a Mecanica dos Fluidos, reparei que sao feitas muitas aproximacoes
para a classificacao dessas substancias (ex.:fluido newtoniano, fluido ideal, etc. Algo intrigante é
o fato de sempre ter como unico exemplo de plastico ideal a parafina... Na natureza nao ha
outros materias que possam ser considerados como tal? Tenho lembrancas de ja ter ouvido
comentarios de que o carbono tambem apresentaria caracteristicas de um plastico ideal... Isso é
fundamentado?

O que quer dizer gap de energia?   R

Há uma polêmica quanto a prova de vida primitiva em Marte motivada pelo encontro de um
meteorito que seria proveniente daquele planeta e que mostra indícios dessa vida primitiva.
Como é possível saber que aquele meteorito encontrado na Antártida tenha vindo de Marte há
milhões de anos atraz?   R

Sou professor de Física e afirmo que a água somente pode sofrer sublimação quando submetida
a temperatura e pressão inferiores às do ponto tríplice (aproximadamente 0,05°C e 0,006 atm).
No entanto vejo em sítios de Meteorologia, livros de Geografia e até em vestibulares a informação
de que a formação de neve e de geada deve-se à sublimação da água. Parece-me absurda tal
afirmação, pois não está com certeza sendo atendida a condição p < 0,006 atm. Estou errado?   R

Porque alguns resistores apresentam um anel preto após o anel de tolerância (que normalmente
é o último)?   R

Cerâmicas supercondutoras, quando resfriadas, impedem a penetracão de campos magnéticos


externos, mas permitem que elétrons fluam por ela sem resistência. Minha dúvida: Para onde vai
o campo magnético dos elétrons que fluem na cerâmica super-resfriada? Anéis de cerâmica
contendo (ou *armazenando*!!) uma corrente elétrica são eletroímãs?

Assisti a uma pequena reportagem (no Fantástico) sobre o universo, e na reportagem falaram
que o universo é finito e que seria semelhante a uma bola de futebol. Poderiam falar algo a
respeito?   R

O que exatamente Einstein quis dizer como: Deus joga dados com o universo.   R

Gostaria de saber se as lentes fotocromaticas são, também, indicadas para usuarios de


computador ? ela protege contra a luminosidade do computador?   R
Alguém poderia explicar como funciona o estilingue gravitacional que as naves usam para ganhar
velocidade para chegar aos outros corpos celestes? Eu imagino que estas naves ganhem
velocidade por conta da gravidade do planeta quando se aproximam, mas quando elas se
afastam, esta gravidade não age com efeito contrário, diminuindo a velocidade que foi ganha?   R

Como através da Geometria Descritiva Gaspard Monge conseguia fazer Napoleão ganhar as
guerras?   R

Não sei se é possível haver explosões no núcleo da Terra como acontece no filme The Core, mas
o que eu gostaria realmente de saber é se estas supostas bombas (iguais as do filme)
conseguiriam explodir em um lugar com uma pressão tão esmagadora?   R

Como se dá o tunelamento através de barreiras?   R

Por que a velocidade no ponto de contato do pneu com o solo é zero?   R

Que especie de harmonicos resultam nos instrumentos cônicos e porquê ? Esta pergunta vem na
necessidade de estudar harmónicos em saxofone.   R

Qual é a estrela mais próxima da Terra? E a galáxia? É a alfa do Cenfauro? Ou a Eta Carina (que é
uma estrela ou uma galáxia?)? Qual é a distância? Tenho um telescópio de 60mm de objetiva e
700mm de distância focal (aumento máximo eficaz de 120x aproximadamente). O que posso ver
com ele? Apenas planetas (já vi marte, saturno, posso ver júpiter, urano, mércurio e plutão?)ou
estrelas também?   R

Sabemos que um dia não tem exatamente 24 horas, isso é um mero arredondamento... E por isso
temos os anos bissextos: passados 4 anos, o "tempo perdido" por esse arredondamento soma
um dia. Agora, a pergunta: por que, passados 2 anos, não temos uma inversão do dia, ie, o dia se
torna noite e vice-versa? Afinal, teriamos acumulado 12 horas... Isso me parece um sofisma, mas
não soube como resolvê-lo. Pensei também em provar que isso não ocorre pela lei de Kepler que
diz que os planetas varrem áreas iguais em periodos de tempos iguais, ou seja, a Terra vai estar
rodando ao redor do sol mais rapidamente em um ponto que em outro e isso compensaria... Mas
não me parece fazer muito sentido. Alguem tem uma soluçao?   R

No filme Contato, de Carl Sagan, aquela máquina onde a Jodie Foster foi lançada me pareceu
muito com a estrutura de um átomo. Sendo que aquelas barras que rodavam faziam papel dos
elétrons, daí pensei: será que o mesmo efeito, o da criação do campo magnético de imensa
proporção, não é produzido também no interior do átomo e por ser tão pequeno nós não o
sentimos?   R

Li que a radiação vermelha é a mais calorífica. Como é possivel se a violeta é a mais


energética?   R

O que interfere na forma física dos seres humanos? Os supostos ETs teriam formas diferentes da
nossa por quais circunstâncias?   R

Pela relatividade restrita, o tempo para um corpo em uma certa velocidade passa mais devagar
do que um parado. É uma questão de referencial essa minha. Uma nave sai da Terra com certa
velocidade (0,9c), para a Terra, a nave está se afastando a 0,9c, mas para a nave, a Terra está se
afastando a 0,9c. Porque os efeitos relativísticos se aplicam a nave e não a Terra? Muitos
tentaram me responder que a nave começou a aceleração para chegar a tal velocidade, porém eu
creio que a matéria não tenha memória, logo quando chegaram a tal situação, porque o tempo
passa mais devagar para a nave e não para a Terra?   R

Por que o suco gastrico e formado de acido cloridrico e no entanto nao podemos ingerir esse
acido?   R

Para líquidos o aumento da temperatura diminui a viscosidade e para os gases a viscosidade


aumenta. Porquê? Existe algum gráfico que explique esse fenômeno?   R

se o neutron é uma particula sem carga elétrica, afinal de contas, como faz-se para acelerá-lo?   R
Tinha uma geladeira num sítio que não havia energia elétrica e funcionava com um botijão de gás
de cosinha e quando abria a porta, para minha supresa acendia a lâmpada como nas geladeiras
elétricas modernas. Como explicar o acendimento da lâmpada, sendo que a geladeira não tinha
nada de especial para fornecer energia elétrica para a lâmpada; de onde vem esta energia que
acende a lâmpada? Andei pensando que poderia ser de um espécie de TEG (Termo gerador
elétrico) que funcionasse com o calor do gás. Mas durava mais de um ano cada botijão de gás e
o funcionamento era contínuo enquanto o fogo do gás estivesse aceso. (Gostaria de receber a
resposta também no e-mail: carlosfac2@ig.com.br)   R

Como a dimensão proposta por Kaluza e Klein interage com as dimensões com as quais temos
contato costumeiramente (as 3 do espaço e o tempo)?   R

o que distingue o conceito de massa inércia da teoria da relatividade das caracteristicas do


conceito de massa inércia elaborada por newton?

Outro dia vi uma reportagem em um jornal que dizia que algumas pessoas estavam escurecendo
os vidros de seus carros com uma solução à base de "terras raras". No outro e-mail que eu enviei
fiz perguntas sobre lentes fotosensíveis. será que este mesmo método pode ser utilizado (terras
raras)? Pergunto isso porque na reportagem eles afirmavam que se passava uma solução à base
de terras raras nos vidros,que se escureciam quando expostos ao sol e voltavam à cor natural em
ambientes com baixa luminosidade. Só que este processo parece que foi proibido por que
quando os carros passavam em um radar em altas velocidades a luz do flash era absovida pelos
vidros. O que são terras raras? O que poderia ser esta substância? Ela pode ser chamada de foto
sensível? Os óxidos de terras raras têm esta propriedade?   R

O que é o tempo imaginário ? Como ele interage com o tempo real ? Como eles se
interrelacionam com o espaço-tempo ? Com se realcionam com as dimensões do Universo?

Gostaria de saber a explicação para o fato de as ondas eletromagneticas acompanharem a


deformação do espaço-tempo de acordo com o previsto pela teoria da relatividade geral,já que a
própria hipótese de existência de um meio para propagação dessas ondas(éter luminífero)foi
descartada?   R

De que modo especificamente, ou seja, em que ponto a teoria das supercordas unificaria a teoria
da relatividade com as teorias quânticas?   R

Segundo a Mecanica Quantica a natureza não é contínua, ela dá saltos, é quantizada e ela estuda
exatamente as partículas básicas que compõem todo o Universo e seu objetivo seria chegar a
unidade da qual todo o resto deriva. Uma vez eu li num livro de física que esta definição que
explicaria o universo seria puramente matemática, uma expressão, já que a definição de algo
nesta expressão teria que ser explicada por uma teoria e continuaríamos com o mesmo
problema. Se algum dia chegarmos a esta expressão e a natureza é discreta como os circuitos
digitais, o que impede de criarmos um universo(o nosso é finito) em um computador e
conseqüentemente nós mesmos poderíamos ser fruto de semelhante dispositivo?   R

O que são bósons livres?   R

O que vem a ser "Açúcar Invertido" que compõe vários alimentos?   R

A voz de Jesus, por exemplo, ou de qualquer pessoa, perdura infinitmente no ar da Terra??? Ou


seja, o som depois de ocorrido, é perene, eterno???   R

Ola, não sei nem se minha questão cabe aqui. Mas aí vai. Seguinte: Os espiritualistas estão
adorando a idéia das conclusões da física quantica....... quando falam na dualidade
onda/partícula....... e no fato de que o observador interfere no resultado do experimento. Sendo
assim, chegam a conslusão de que a mente e as coisas observadas fazem parte de uma coisa
só.......podendo a mente manipular tudo o que ela quiser........ pois tudo faz parte do "Eu"
cósmico..... o que chamam de Deus. Tenho um amigo (espiritualista) que está tentando me
empurrar atravez da física quantica que, a observação é afetada pela presença do observador,
sendo assim, se o observador pode afetar o movimento de uma particula, significa que o
observador esta criando com a mente aquele resultado. Ou seja, tudo que existe é criado pela
nossa mente, e sendo assim podemos fazer qualquer coisa com o poder dela....... como
desintegrar uma cadeira ou o próprio corpo com o poder da mente. Será que poderiam me dar
alguma explanação sobre isso?   R

Gostaria de saber qual material esfria recebendo energia elétrica. Pois as geladeiras de
automóveis (portáteis) não têm gas comprimido, são acionadas apenas por eletricidade que
chega a um material e este pode aquecer ou resfriar. Que material é esse ou é um
equipamento?   R

Favor, enviar mais informações sobre o PELTIER. Estamos interessados em utilizá-lo como
sistema de refrigeração em unidade de óleo-hidráulica, substituindo os sistemas convencionais
que são utilizados por ar ou água. Por isso o interesse em saber quem fornece,utilização e
propriedades deste.   R

Gostaria de alguma luz sobre a "flecha do tempo". Em muitos livros ou documentários dá-se o
exemplo de um filme onde, digamos, uma chícara cai e se quebra e quando o filme é passado de
trás para frente vemos a chícara se "desquebrar". Acontece que nós não estamos no filme.
Pergunto: se estivéssemos no filme, faria diferença se o filme fosse passado normalmente ou ao
contrário?   R

Qual a relação entre a massa/energia total de uma estrela supernova supermassiva e que
colapsa sem nenhuma perda de massa para o espaço, e a massa/ energia do buraco negro a que
essa estrela da origem? a massa/energia do buraco negro é maior que a massa/energia da
supernova inicial? e por quê?   R

Fiz recentemente um concurso público para minha área - arquitetura - e uma questão me deixou
um tanto curiosa, gostaria, se possível, de submetê-la à sua avaliação, visto que o raciocínio
lógico diverge da resposta dada pela banca do CESPE-UNB, e não encontro uma resposta
satisfatória na bibliografia que tenho à disposição... Eis a questão, cujo gabarito oficial considera
errada, e eu, certa: "Os jardins de inverno que são fechados com vidro funcionam como uma
estufa. A luz solar passa através do vidro e aquece as superfícies internas. Estas emitem raios
infravermelhos longos, que não podem atravessar o vidro de volta." Ok, sei que as superfícies
internas (veja que não especificam o tipo) não "emitem" raios, mas poderiam estas refletí-los, e
estes atravessarem novamente o vidro?   R

Gostaria de saber qual é, afinal, a função da cóclea?   R

O espaço é infinito ou finito?   R

A luz vermelha estimula a cicatrizaçao de cortes porque estimula a mitocondria, organela


responsavel pela produçao de energia das celulas. Existe alguma explicaçao para esse
fenomeno?

Por que cubos de gelo se grudam ao serem colocados próximos uns aos outros?   R

Não sou especialista no assunto, mas se não estou enganado, a teoria da relatividade demonstra
que o universo é finito porém ilimitado. Sendo que se um super-foguete viajasse por zilhões de
anos em linha "aparentemente reta" na verdade estaria traçando uma curva e voltaria ao mesmo
lugar. Minha pergunta na realidade é esta: Existe alguma possibilidade, em tese, que o tempo se
comporte de forma semelhante? Ou seja, que daqui a zilhões de bilhões de anos, tudo volte a se
repetir? E assim sucessivamente? Estaríamos presos em ciclos temporais de dimensões
googolianas?R

Em que consistem as desigualdades de Bell?   R

Uma explosão atômica produz um nuvem em forma de cogumelo de grandes proporções, não
comparáveis, entretanto, aos grandes cumulus-nimbus (CB) produzidos por fenomenos naturais,
como as correntes convectivas e as frentes frias. É possível, então, medir a energia que formam
essas nuvens CB em Megatons?   R

Como podemos determinar o poder de resolução entre ondas de diferentes comprimentos?   R

O que é MOSFET?   R
O que se entende por inclinação magnética?   R

O que acontece quando ao dimensionar uma motobomba, a bitola da tubulação de sucção for
subdimensionada, ou seja, a bitola da tubulação de sucção for menor que a bitola nominal de
operação projetada para a bomba?   R

Existe algum material transparente que seja bom condutor de eletricidade?   R

Gostaria de saber quais são os avanços sobre o Universo Paralelo? Qual seria a melhor literatura
sobre o assunto para leigos, que pudesse explicar a possível existência do Universo Paralelo?   R

Estudando termodinâmica, deparei-me com o seguinte problema: "Todos sabemos que se


colocarmos um cilindro com gás ideal no fogo a temperatura do gás aumenta quando o volume
permanece constante . Logo, a energia cinética média das moléculas aumentou. Se a energia
fornecida ao gás é devida às forças que as paredes do cilindro exercem sobre ele, o gás realiza
sobre as paredes do cilindro um trabalho com o mesmo módulo. Como as paredes do cilindro
não se deslocam, podemos concluir que o trabalho realizado sobre o cilindro é nulo
contradizendo a informação de que as paredes do cilindro realizaram trabalho sobre o gás." O que
está errado nesta afirmação?   R

Como funciona um motor iônico ? Qual sua viabilidade ? ( não sei se devo fazer essa pergunta a
um químico ou engenheiro).   R

Quais os fenomenos que provocam o estalar de um chicote?   R

Gostaria de saber qual é o gás que existe na tubulações dos aparelhos de ar condicionado. Se
estas tubulações romperem e o gás vazar para o ambiente, qual o perigo para as pessoas que
estão dormindo?   R

A pelicula fotocromática sendo depositada em outro material transparente, não sendo este uma
lente, teria a mesma eficácia?   R

O que são partículas virtuais, e o que fazem no redor dos atomos, ja que tem vidas tão curtas?   R

No paradoxo de Schroedinger é proposto um mecanismo para se testar o efeito da equação de


onda da mecânica quântica. Como comprovadamente não podemos ter um gato meio morto fica
claro que algo ocorre que viola a equação, já vi muitas propostas, entre elas a existência de
mundos paralelos, porem todas as propostas se baseiam no colapso da equação devido a
presença de um observador. Existe um modelo capaz de resolver o paradoxo com grande
certeza? Se realmente o fato de se observar o sistema causa o colapso, então a consciência é
uma entidade elementar por estar influindo no mundo quântico (e isto implica em varias outras
conclusões) ou é o que sempre pareceu, um efeito da complexidade da matéria viva?   R

Qual a máxima freqüência possível para uma onda eletromagnética? Existe uma classe de ondas
mais energética que a radiação gama?   R

O que é entrelaçamento quântico de partículas?   R

Como funciona a holografia? Será que um dia teremos imagens holográficas projetadas no ar?   R

Gostaria de saber as diferentes aplicações e utilidades das antenas com polarização


circular(direita ou esquerda) , linear (vertical ou horizontal) e eliptica?

Certa vez observei um fenômeno que me intrigou e por isso busco uma resposta. No interior uma
senhora estava fazendo um doce de leite. Quando o leite ia ferver eu a avisei que o mesmo iria
derramar. Ela falou que não e pôs um pires emborcado no leite, o leite ferveu até quase secar
sem derramar uma gota. Qual a explicação física para tal fenômeno?   R

Existe a possibilidade de se fabricar por meio semi-artesenal as lentes fotosenssiveis? E


como?   R

Pessoal, uma bala disparada de uma arma apontada para o alto é tão perigosa quanto uma bala
disparada totalmente na "horizontal"? Acho que a quantidade de movimento de uma bala
disparada para o alto se conserva, certo? Então ela chegaria ao solo com a mesma velocidade
com que partiu da arma? E seu poder destrutivo diminuiria ou continuaria o mesmo?   R

Como o uso de laser atua no tratamento de traumas, fraturas...   R

Há algum aparelho especializado em provocar microfonia (barulho estridente provocadopela


proximidade do microfone próximo à caixa de saída do som)à distância? Ou, como posso
provocar a microfonia estando a uns 100 m da aparelhagem de som?   

Qual a possibilidade de se contruir um elemento propulsor para um aeromodelo, utilizando o


principio do tubo de venturi?

Gostaria de saber se gases podem ser separados por eletrólise? Algum cientista conseguiu
captar gases a partir da atmosfera? Certa vez eu ouvi que um cientista israelense havia criado
uma máquina de produzir água captando os gases da atmosfera. Encarecidamente peço que me
enviem maiores informações sobre esse cientista e suas pesquisas.   R

para que serve o colorido azulado existente no para brisas de alguns carros como no renault
megane, e porque é que só se consegue ver esse colorido do lado de fora do carro para dentro
?.Poderá ser prejudicial a quem se encontra no interior do veículo em presença da luz solar?   R

Por que, no movimento de rotação de um corpo rígido em torno de um eixo fixo (passando por
seu eixo de simetria), a velocidade angular (e conseqüentemente, o momento angular) do corpo é
um vetor perpendicular ao plano de rotação (paralelo ao eixo de rotação)? Se a velocidade
angular (w) é a derivada do deslocamento angular (fi) em relação ao tempo, logo w tem a mesma
direção de fi (pois os dois são vetores). Não é "lógico" (para mim) que o vetor fi aponte
perpendicularmente ao plano de rotação, já que não há delocamento angular nesta direção.   R

Em uma linguagem simples, o que são tensores? O que a teoria de Einstein tem a ver com
tensores?   R

GOSTARIA DE SABER O QUE É O "PLASMA", E COMO FUNCIONAM OS NOVOS APARELHOS DE TV


QUE TRAZEM ESTE MATERIAL EM SUAS TELAS.   R

Existem no mercado, já a muitos anos, colchões com ímãs que geram no colchão um campo
magnético de aprox. 800 gauss e mais recentemente, lançaram novos colchões, adicionando
emissores de raios infra vermelhos longos. Existe a possibilidade de um interferir com o outro
gerando uma terceira resultante da combinação entre estes dois elementos ou funcionam de
forma independente, cada um oferecendo seus próprios benefícios?   R

Perguntei a voces sobre a tão falada forças e referenciais não-inerciais e como um corpo de
massa de 10 kg poderia puxar outro de 100kg, e qual seria o princípio físico que explicaria isso.
Só para esclarecer, estou me formando em economia, sendo, portanto as minhas perguntas
meramentes por curiosidades... Por favor, assim que possível respondam-me... Obrigada.   R

Preciso fabricar um mecanismo de luz polarizada para analisar peças de plastico (polipropileno).
atraves da emissao de luz polarizada na peça de plastico, pode-se ver a tensao residual do
material injetado. como poderei construir um mecanismo destes? que tipo de lente se utiliza?
onde sao vendidas estas lentes? qual luz deve ser utilizada? monocromatica?   R

estou lendo o cavalo de troia de J.J.BENITEZ e fiquei intrigada sobre a particula swivels, eu vi
recentimente em uma reportagem jornalistica algo sobre a descoberta de uma particula super
minuscula que feita se eu nao me engano da difusao dos atomos , nao seria esta particula
swivels, gostaria demais informacoes, obrigada monica   R

Porque as densidades de energia de campo magnetico e eletrico sao iguais?

Um professor de física me explicou que, um corpo, ao se aproximar da velocidade da luz vai


ganhando massa inercial, e não massa de "quantidade de matéria". Qual a diferença entre
elas?   R

Eu trabalho em uma firma eletromecanica, fazendo reparo de motores eletricos de indução, e


gostaria de saber como eu faço para descobrir qual é a potencia deles sem ter que ver em suas
plaquetas. Inicialmente eu pensei na formula da potencia (P = V.I), só que ao colocar carga no
eixo desse motor a corrente ira aumentar e talvez me e uma leitura errada de sua potencia. Entao
eu gostaria de saber se existe outra formula que eu possa usar. Muito Grato.   R

Qual deve ser a temperatura no centro do Buraco Negro? Se a matéria no seu centro é
extremamente condensada, ela deve ser grande por conta da pressão exercida ou deve ser
pequena por conta do pouco espaço que a matéria tem para vibrar?   R

Eu gostaria de saber se há registros de terremotos nos polos da terra?

Gostaria de saber por quê um submarino feito de paredes de aço só pode atingir uma determina
profundidade, se não ele arrebenta devido a pressão. E em profundidades bem maiores vivem
peixes e outros animais como se estivessem na flor d'água.   R

É possível que dois ou mais planetas *habitem* uma mesma órbita?   R

É verdadeira a afirmação de que a água oxigenada não deve ser empregada em curativos? Que
estudos demonstraram que seu uso poderá ser mais prejudicial que benéfico?   R

Boa Tarde ! Gostaria de saber como surge um arco-iris em forma de circulo , ou seja em volta do
Sol . Como por exemplo o arco-iris que observei hoje 30/12/2003 às 11:30hs . OBS.: Eu estava na
av. 23 de maio quando observei esse fenomeno.   R

Pergunta relacionada ao comentário de Luiz Ferraz Netto sobre a pergunta de Rodrigo


Coutinho/RJ que diz: "Temos na Terra dois corpos um maior que o outro em cima de uma
balança de braços equilibrados na posição horizontal. O que aconteceria com a balança se
levássemos essa experiência para a Lua?", o comentário diz que: "...balança penderá para o lado
onde está o 'corpo maior'...". Minha pergunta: De acordo com o exposto, na Terra os braços da
balança estão na horizontal (em equilíbrio) indicando o mesmo peso. Como o corpo maior tem a
ajuda de empuxo maior devido ao maior volume isto parece indicar que sem a presença de ar
(empuxo) o peso do corpo maior seria menor que o peso do corpo menor, ou seja, sua massa é
menor, a balança só está em equilíbrio pela ajuda de um empuxo maior. Neste caso, na lua, sem
ar (conseqüentemente sem empuxo), a balança não deveria pender para o lado do corpo menor?

Anteriormente a grande diferença entre um motor a gasolina e outro a álcool não estava apenas
na riqueza da mistura ar/combustível como também na pressão interna do motor. Motor a
gasolina tinha pressão mais baixa ao passo que os motores a álcool mais alta, e este aumento de
pressão é que aumentava a potência do carro a álcool. A gasolina usava uma pressão mais baixa
para não haver detonação a "explosão antes da hora". E o álcool usava uma pressão mais alta
para conseguir "queimar" o combustível na câmara de combustão. Como fica, então, a pressão
nos motores duplo-combustível? Não deveria haver detonação quando utilizado só gasolina ou,
então, haver "engasgamentos" quando utilizado apenas álcool (ou uma mistura mais rica em
cada um dos combustíveis para cada caso)?   R

Se as partículas de segunda e terceira geração decaem tornando-se partículas de primeira


geração, por que então elas são consideradas fundamentais?   R

Gostaria de saber se a ingestão de água imantada é realmente benéfica ao nosso organismo e


quais os benefícios da placa de infra vermelho longo no interior do aparelho imantador.   R

Como eu posso bloquear as ondas eletromagnéticas (geradas por um gerador de alta


frequência), para evitar que elas interfiram em aparelhos eletrônicos? Eu posso por exemplo usar
uma "gaiola de Faraday" para revestir a fonte, se sim, como posso e com o que eu faço essa
gaiola?   R

Se eu quiser ter todos os meus eletrodomesticos sem fio, eu poderia colocar em minha casa uma
bobina de Tesla? Qual seria a adaptação que eu deveria fazer nos aparelhos para isso? Por que
isso não é utilizado comercialmente?   R

GOSTARIA DE SABER COMO FUNCIONA O QUARTZO NOS RELÓGIOS DIGITAIS E


ANALÓGICOS.   R
Um dos fenômenos mais perigosos para a aviação é a formação de gelo nas estruturas das
aeronaves em vôo. Este gelo é formado a partir de água líquida (nuvem) super-resfriada que é
explicado pela sobrefusão. No entanto, isto só é possível se a água estiver em repouso. A minha
pergunta é: como a água pode estar em repouso ou em escoamento laminar quando os ventos
em altitudes elevadas podem chegar a velocidades bem acima de 60Kt?   R

Uma partícula beta é chamada de elétron. Ela é um elétron? Se sim, por que aparece em reações
nucleares e não em químicas? O fato do número atômico (Z) da partícula beta ser -1 é uma mera
convenção? Se altera o Z no elemento resultante, existem elétrons no núcleo?   R

Gostaria de saber o que significa quando dizem, por exemplo, "100 mm de chuva". Qual o
significado dessa quantidade?   R

Gostaria de saber que se, deixarmos um copo com uma bebiba alcoólica no Sol, o que evapora
primeiro: a água ou o álcool? Ou os dois evaparam ao mesmo tempo (estão misturados)?   R

Quando observamos galaxias cada vez mais distantes estamos observando o espaço cada vez
mais primordial, ou seja, estamos observado o passado do universo. Daí a pergunta: Pelo menos
conceitualmente, poderíamos observar a história evolutiva da nossa própria galaxia, desde sua
formação?   R

Gostaria de saber qual a temperatura do fogo (chama) que sai na "boca" do fogão. É a mesma
temperatura da chama do isqueiro? Como os fornos industriais conseguem atingir temperaturas
tão elevadas?   R

Num sistema com unicamente dois corpos orbitando, quando há perda de energia os corpos se
afastam ou se aproximam? Dá para explicar um pouco?   R

Os bósons fundamentais também possuem sua antipartícula correspondente?   R

Como posso eliminar o eco de uma ambiente de aprox. 5mx5m (garagem quando está sem o
automóvel)?   R

Gostava de responder à minha filha de 15 anos pelo que peço a vossa ajuda. Temos os conceitos
de um modo simples e superficial sobre velocidade, acelaração, gravadidade, órbita.... Eu cada
vez menos Ela cada vez mais com o passar dos anos. Perguntou-me como é que um austronauta
sai da nave (viajando no espaço) e está parado em relação à mesma mas a nave está a viajar a
uma velocidade muito grande em relação à Terra.   R

Pode existir um arco-íris formado por uma outra radiação que não fosse a luz visível?   R

Existe diferença na sensação de frio entre as raças? Ou seja negros sentem mais frio que
brancos?   R

Gostaria de saber qual vai ser o destino das sondas Voyager I e II. E também: elas ainda
funcionam? Se funcionam, como isso é possível?. E mais: qual o segredo para essa missão ter
tido tanto êxito?   R

Eu gostaria de saber por quê o programa espacial Brasileiro não consegue êxito. O que falta em
nossos foguetes? Falta mão de obra qualificada? Ou o motivo é outro? Como, por exemplo,
investimentos?   R

Gostaria de saber se existe isolante para campo magnético. Se existe, gostaria de saber quals os
materiais mais utilizados e os mais eficientes.

Sou brasileiro e atualmente resido na Itália. Caminhando extensivamente, por este país, notei que
os portos daqui possuem uma água límpida, cristalina, onde se vêem cardumes de peixes e as
pessoas nadam no verão, junto a embarcações, sem temer por sua saúde. Estes portos, alguns
com mais de 2000 anos, possuem um movimento intenso de embarcações que entram e saem a
todo momento. Fico intrigado comoas águas da baía de guanabara, no Rio de janeiro - um porto
relativamente jovem se comparado aos daqui - são tão sujas (para não dizer imundas)a ponto de
nada sobreviver ali. O que acontece com o sistema de controle de poluição, que não consegue
manter as águas limpas como, por exemplo, as da baía de Napoli.Questão: Existe algum
processo químico ou físico para limpar as águas da baía de guanabara???. Não vi, por aqui,
nenhum processo dispendioso que justificasse a displicência com que tratam um dos cartões
postais da cidade do Rio de Janeiro. Questão: O que eles fazem, por aqui, para manter as águas
tao limpas, a ponto de vermos o fundo do porto, mesmo com cargueiros e outras tantas
embarcações a entrar e sair do porto a cada minuto?   R

Trabalho com som, e tenho muito problema com microfonia, quais dicas devo seguir para que
esse "barulhinho irritante" desapareça da minha vida?   R

Gostaria de saber informações sobre o sistema de comunicação e alimentação da sonda


espacial spirit, para trabalhar com os alunos em sala de aula.

Qual o melhor gas que posso usar em um envoltorio(balao) para produzir um empuxo que
neutralize o peso total de um conjunto (objeto + balao) e que volume deste gas eu preciso para
neutralizar(nao precisa subir) cada kilo deste conjunto?   R

Como funciona a solda elétrica ? De que é composta as varetas de soldas e porquê? Em relaçào a
amperagem eu já sei, mas porque eu não levo choque se eu pegar o cabo de solda e no "terra" já
que a máquina trabalha com corrente alta?   R

O interior do aparelho de micro ondas da casa da minha namorada tem um furo relativamente
grande de modo que se dá até mesmo para notar a parede da cozinha, e mesmo assim eles
continuam a utilizar o aparelho, estou preocupado com essa situaçao, e preciso de informaçoes
se há risco pra saúde e quais são.   R

Como faço pra medir a temperatura de um mosquito?   R

Caso ja tenham ouvido falar da grande explosao siberiana em 1908, se é que de fato ela ocorreu,
qual a opiniao de vocês?   R

Quando um raio cai no mar, os peixes morrem eletrocutados?   R

Gostaria de saber mais informacoes sobre o decimo planeta do sistema solar.   R

Encontrei esse site numa busca que fiz para descobrir como Avogadro chegou ao "número de
avogadro", pois necessito disso para um trabalho de química... c possível gostaria de uma
resposta p isso... n eh p fla o Q eh o nº d avogadro, pq eu SEI eu kero saber eh COMO ele chegou
a esse número... + naum eh essa a minha pergunta.. eu, aluno do 2º ano do E.M. e que pretende
prestar vestibular para direito, gostaria de saber: Qual a útilidade dessas coisas cheias de
números pra mim? Como eu poderia utilizar equações de velocidades, ou vetores, na minha vida,
após me formar? Não que eu ache essas coisas inúteis para o mundo, + eu, qto + tento, menos
entendo a utilidade delas para MIM! Me expliquem, como a física que me é ensinada poderá
SERVIR para algm como eu (futuro estudante de uma área de Humanas) Grato pela compreensão
e paciência!   R

Gostaria de saber se caso uma linha elétrica de alta tensão que cruza um rio, caísse no mesmo,
qual a possibilidade de uma pessoa que esteja tomando banho no rio no momento da queda do
cabo de energia, levar um choque.Existe um limite de propagação do choque na pessoa, ou se ela
estivesse mais distante levaria também o mesmo choque!

Queria saber um pouco sobre a experiência dos "Hemisférios de Madgeburgo".      R

Quais são os novos elementos químicos da tabela periodica, eu gostaria de saber a descoberta,
nomenclatura e importância.

Por que quando deixamos o pacote de biscoito aberto os biscoitos amolecem e quando
deixamos o pacote de pão aberto o pão endurece?

Uma vez ouvi o Cesar Lattes falando sobre uma teoria de que todas as constantes físicas variam
com o aumento da idade do universo... Alguma bibliografia?   R

fala-se muito atualmente em se enviar uma missão tripulada ao planeta Marte. Mesmo sabendo
que a velocidade de escape de Marte é 5 km/s e a da Terra é mais que o dobro (11,2 km/s),
haverá condições de uma nave tripulada partir de Marte para Terra?? Quais as relações
energéticas envolvidas??

1)Se uma pessoa chegasse a velocidades próximas à da luz ou de um buraco negro,a mesma
viajaria ao passado ou ao futuro?seu fluxo de tempo aumentaria ou diminuiria?(POR
FAVOR,expliquem-me detalhadamente,para que eu possa entender "isto" melhor) 2)Lí em uma
edição da Galileu sobre viagens no tempo que os 14 satélites do sistema GPS que os mesmos
deslocam-se à uma velocidaede de 14.000Km/h,causando desta forma forma uma distorção no
espaço-tempo de 45 microsegundos por dia(de adiantantamento). Suponhamos que estes
continuem movendo-se durante 100 anos em velocidade constante,os mesmos estariam
praticamente no futuro(em outras palavras,outra dimensão?...)e nós não saberíamos de sua
existência,apenas teríamos uma lembrança de nossos viajantes,certo? 3)(e última)Sabendo
disto,como Stephen Hawking diz haver o sistema de proteção cronológica do tempo?

Trabalho numa clínica de fisioterapia, onde usamos muito aparelhos de ondas curtas. Queria
saber se este tipo de aparelho pode fazer mal aos profissionais que o manuseiam e em que
intensidade pode prejudicar uma mulher grávida de mês que o manuseia diariamente. Grata pela
atenção.   R

Queria saber como posso contruir um gerador de baixa capacidade para q ele forneça energia
para uma maquete... a hidor energia é minha melhor fonte no local... Seria possivel a construção
de uma pequenina usina hidro-eletrica para abastecimento das luzes da maquete?   R

Se a terra gira a uma velocidade alta, então porque não estamos em constantesm e fortes
ventos? a não ser que a atmosfera gire junto com a terra. E se gira, prque? já que quando giramos
uma esfera qualquer, o ar não gira junto.   R

Há alguma diferença nos termos banda de passagem e banda passante? nesse caso, o que
significa cada um?   R

Existe alguma explicação na física quântica para o fenômeno, chamado pelos químicos de
*valência*, ou seja, a *necessidade* de um átomo preencher seus orbitais externos, até o ponto
de se criar íons? Não consigo atinar com qual tipo de força se manifesta nesse fenômeno.   R

Existe hoje uma grande discussão de que sons de baixa frequência (50 a 200 Hz) e intensidades
de 200 dB (calculados no mar que é diferente do ar) emitidos na água do mar pode causar
fissuras nos ossos de mamíferos marinhos (baleias e golfinhos). Após ler o artigo sobre
ressonância, entendi que para que isso ocorresse os ossos teriam que vibrar nessa mesma
frequência e em amplitude que afetasse a elasticidade do osso. Isso é realmente possível de
ocorrer? Como eu poderia realizar um experimento que me mostrasse esta hipotese?   R

Por que ao colocar um gelo em um recipiente cheio de óleo vegetal, ele fica se movimentando
constantemente?   R

Gostaria de saber quais os tipos de lâmpadas que geram raios UV e as que não geram raios
UV.   R

É possivel criar um aparelho para que o ser humano consiga enxergar as ondas eletromagnéticas
existentes no ar?   R

Gostaria de saber,qual é o produto que se mistura ao gás hélio para dar cheiro ao gás?

Estou realizando a experiência do telefone de fio, usei barbante de diversos tamanhos e fio de


naylon de diversos tamanhos. Pude constatar, que com o barbante obtivemos melhores
resultados. E também pude observar, que usando lata de aluminio (nescau) e barbante, o som
ficou mais nitido e mais alto. Como podemos demonstrar através de equações matemáticas?   R

Eu queria saber quem inventou o chiclete e toda a história dele.   R

Oi, sou professora de Biologia, Ciências e também trabalho Química e um aluno questionou, na
aula de Ebulição, evaporação, que estávamos estudando que a 100ºC a água ferve, ele perguntou
quando que os mares e rios começam a evaporar, a que altura?? Sei que eles evaporam
diariamente, mas gostaria de uma resposta científica. Conforme a temperatura aumenta a
evaporação aumenta, mas a que temperatura começa a evaporar?   R

Quisiera tener una respuesta practica de porque en un ciclo simple de vapor, por ejemplo, el vapor
que sale de la turbina no se lleva en forma directa a la caldera, sin pasar por el condensador y la
bomba? entiendo que esto violaría un principio termodinámico (siempre se debe rechazar calor)
pero en la practica que me lo impidiría?   R

Porque a água do mar faz espuma? Tem a ver com gases ou partículas que nela se
encontram?   R

eu estava assistindo a Cultura e vi os físicos falarem do computador quântico *um super


computador*, vcs acham que exite uma possibilidade de ele ser explorado aqui no Brasil?   R

Um fato interessante tem chamado minha atenção e de meus colegas, não bebo e nas baladas
só bebo agua tônica, acontece que a agua fica azulada (a água é sevida em copos de vidro
transparentes e as luzes do bar que frequento são luzes ditas frias). Fica muito bonito e chama
atenção - fica um drink exótico mas é só agua tônica. Por que a a´gua tônica fica azulada?   R

Apenas sobre-tensão queima aparelho eletrônico ou sobre-corrente também? Se eu ligar um


aparelho eletrônico a um transformador de 1,2A a 24V ele queimará se sua característica de
fábrica é de 0,5A a 24V?   R

Por que os mergulhadores tem de fazer uso da descompressão quando mergulham a certa
profundidade? Vi na tv que no nordeste os lagosteiros não descomprimem e estão ficando
doentes, gostaria de saber os motivos da descompressão e desta doença em quem não
descomprime.   R

Segundo Planck a energia é quantizada. Gostaria de saber se existem exemplos na Física


Clássica onde também ocorra a quantização da energia.   R

O nêutron é formado por três quarks ou por um próton,um elétron e um neutrino?   R

Quais as diversas opções existentes no mercado, para se controlar velocidade e torque de um


motor trifásico?   R

gostaria de saber mais sobre energias acumuladas no corpo, pois trabalho com lixamento de
magnesio placas de computadores e celulares, entao as minhas roupas grudam demais em meu
corpo, e nao consigo me livrar, pois aqui no japao eu seco as roupas na lavanderia e quando as
tiro da maquina estao todas grudando e so puxando comecam as estrala e os pelos dos bracos
ficao tudo levantado o que posso fazer para eliminar isso.   R.

É possível construir um fractal 3D com um tetraedo se em cada face do mesmo for adicionado
um novo tetraedro cujas faces tenham uma área igual a 1/3 da do primeiro e sobre esses 4 novos
tetraedros o processo for repetido e assim por diante ad infinitum?   R

Existe algum processo conhecido (ou em desenvolvimento) de obtenção de hidrogênio com alto
grau de pureza (>97%) a partir do ar atmosférico?   R

Porque quando abrimos imediatamente a geladeira pela segunda vez, a porta fica difícil de
abrir?   R

O Universo não pode ser evolutivo de tal forma que num determinado ponto ele cria outro dentro
de si próprio e, nesse mecanismo recursivo a noção de tempo poderia ser completamente
diferente de um para o outro?   R

Pelo que eu tenho lido aqui no site, na TQC as partículas massivas só interagem através dos
bósons fundamentais. Então ao se aproximar cargas carregadas eletricamente estas enviam
uma a outra fótons e então haverá atração ou repulsão. Gostaria de saber quem define e/ou se o
fóton é o mesmo para os dois casos e como as partículas “sabem” que estão sofrendo atração
ou repulsão e o módulo da força (imagino que o módulo depende de quantos fótons atingem
cada partícula)?   R
Como as ondas gravitacionais de outros planos espaço X tempo podem afetar a órbita de
planetas?

Sendo o universo infinito,porque os astronomos disseram que ja descobriram 10%?   R

1) Você poderia dar exemplo de outros tipos de bósons além dos mediadores que possam ser
usados na produção do Condensado de Bose-Einstein? 2)No caso do Condensado Fermiônico
que tipo de férmion é comumente utilizado?   R

Preciso retocar a pintura interna e externa do meu forno de micro onda. Qual são os tipos de tinta
que posso usar sem riscos para a saude e o bom desempenho?

Somos fabricantes de lentes oftlálmicas e temos em nosso processo de produção uma lavagem
de lentes por imersão em solvente orgânico.Essa imersão sendo submetida a ultra som a
limpeza será mais eficiente e com menor atuação humana. Como faço para meu líquido cavitar?
Ou se vocês conhecem algum equipamento para volume de 10 litros?

Gostaria de saber o que é exatamente o fogo de Santelmo e se é possível reproduzí-lo em


laboratório?   R

Gostaria de saber o que vem a ser batimento de frequência em sistemas de rádio comunicação,
como ocorre e o que deve ser feito para se reduzir ou eliminar seus efeitos.   R

Gostaria de saber porque os americanos são tão persistentes em não aderir a unificação dos
Sistemas de Unidades (SI)?   R

O que é um buraco de minhoca? Como ele se forma?   R

O laser infravermelho usado em odontolgia como sendo de baixa frequencia (780 nM e 70mW) é
adotado cada vez mais em diversas patologias, este mesmo laser tem aplicações para
dermatologia?, em quais casos? e qual sua potencia?

Parabéns ao Prof. Omar Rodrigues Alves - CEFETSP - São Paulo - SP, pela brilhante explicação
sobre o sistema de refrigeração por absorção. Sou Engenheiro Eletricista e gostaria de saber se é
possível (ou viável) construir pequenos sistemas de absorção para serem utilizados no
resfriamento de equipamentos elétricos (painéis, transformadores, computadores)?   R

Gostaria de saber as dimensões e perímetro das muralhas de Jericó, e também a profundidade


média do leito do rio Jordão em Israel.   R

Eu li em uma revista que a fórmula da água teria sofrido alteração, isto procede?   R

É arriscado abrir um display de cristal líquido? Ele é tóxico?   R

A imprensa divulgou (com sensacionalismo e pouca informação coerente) que astrônomos


teriam identificado um corpo celeste composto quase que unicamente de Carbono (que
prontamente foi batizado de "Estrela de Diamante"). Perguntas que me ocorrem a respeito: 1 - por
que diamante, não grafite ou Buckenmeisterfullerene? 2 - a existência de um corpo celeste dessa
natureza não tornaria mandatória a revisão de tudo que se calculou quanto à formação dos
elementos "pesados" (além do Hélio) dentro das estrelas?

Preciso construir um psicrômetro de solo. Para isso vou usar termopares para determinar a
temperatura do ponto de orvalho. A minha é dúvida é como equacional o problema. Quando for
aplicada uma correne elétrica uma das junções vai esfriar e o vapor d´água ira se condensar
sobre a mesma formando orvalho (temperatura do ponto de orvalho). Como conhecer a
temperatura do Ponto de Orvalho (PO)? Como quantificar a energia fornecida?   R

queria saber porque quando inalamos hélio de um simples balão, nossa voz é modificada, fica
estridente, distinta da original?   R

Eu queria saber a origem do campo magnético terrestre. Eu li nessa seção que esse campo se
origina do movimento do núcleo líquido de ferro e níquel. Mas campo magnetico é criado por
cargas em movimento. O nucleo nao é eletricamente neutro? Se é neutro de onde vem o campo
magnético?   R

É possível determinar se uma palavra foi escrita por uma determinada caneta ou se numa
palavra, à qual foram acrescentadas letras, é possível determinar-se que as tais letras foram
acrescentadas pela mesma caneta? estou me referindo a caneta do tipo esferográfica.   R

Gostaria de saber de que forma os raios infravermelhos longos atuam na redução do ácido
láctico durante os exercícios?   R

Como surgiu e quais os métodos de se fazer uma marcação isotopica?

Em tempos antigos as janelas dos aviões tinham ângulos retos em seus cantos, mas descobriu-
se que a partir destes cantos surgiam trincas que provocavam ruptura das estruturas. Hoje as
janelas tem seus cantos arredondados e as trincas desapareceram. Como a Física explica este
fato? Existe alguma lei ou teorema que rege este fenômeno?   R

Sou veterinário e atendi um cão que havia engolido uma válvula de panela de pressão. Gostaria
de saber qual é a composição (ligas materiais) que compoem a válvula da panela de pressão.   R

Olá ! Eu estava usando minha máquina digital em um aniversário e a estava "guardando" em cima
de um forno micro-ondas, que estava sendo usado na festa, vez por outra pegava a máquina,
tirava algumas fotos e colocava novamente em cima do micro-ondas. No outro dia, para a minha
surpresa, não existia nehuma foto armazenada na memória flash. É possível que se o forno
estivesse com vazamento, "apagasse" as fotos sem destruir a memória ? Porque fiz um teste e a
memória está funcionando normalmente.   R

Gostaria de saber se a gravidade existe segundo a relatividade geral.   R

qual a definicao, os sinais, sintomas e o tratamento no caso de deslocamento da retina?   R

Sou moradora de um apartamento no 6º andar, e há 6 meses ouço uma espécie de ruído


constante e insuportável, como se fosse um zumbido agudo grave que causa dormência na
cabeça e dor de ouvido. Vale dizer que há antenas de operadoras de celular numa distância muito
próxima do meu prédio. O que pode estar acontecendo?   R

Por que o onibus espacial entra na atmosfera terrestre em alta velocidade? Nao é possivel atingir
uma velocidade segura para a reentrada, ou seja, frear uma nave no espaço?   R

Como anular (diminuir) a freqüência de uma máquina não enclausurada, que em seu processo de
solda a frio (alta freqüência), emite ondas de 20MHz, o que acarreta interferência em aparelhos
de TV, rádio e telefone sem fio?

Li em algum lugar que elétrons e Quarks possuem spin de 1/2 e que, portanto, necessitavam de
"duas voltas completas para retornar ao seu estado inicial", entretanto, o que se quer dizer com
isso? Que "estado" inicial e intermediário são esses? Estariam relacionados à dualidade onda-
partícula ( além do mais, expliquem-me o que significa isto, porque, dizer secamente que é o
comportamento do elétron como onda e partícula não é válida)? Se massa relativística é
equivalente à energia e elétrons são quantas de energia, de que maneira eles podem ser
partículas, ou seja, como relacionar essa energia quantizada`à massa de repouso?   R

Na pergunta sobre a igualdade E = mc² = hf, o Daniel Ferrante dá um link para uma outra resposta
dele, no site da USP, sobre o assunto. Lá, ele levanta uma questão que iria ser objeto da minha
pergunta: e para uma partícula com massa de repouso zero? E ele explica: aí, vale E = hf. Tudo
bem: isso é um dado experimental largamente comprovado. Mas, para onde vai a matemática?
Um fóton automaticamente ganha uma "massa" m = E/c², tanto que ele sofre e provoca efeitos
gravitacionais, e "acelera" instantaneamente de zero a "c". Como eu não tenho notícia de que
regiões do espaço onde não haja massa estejam a emitir fótons (ou será que Hoyle estava
certo?), só posso concluir que a equação E = mc² seja descontínua para m = 0. Certo, ou não é
bem assim...?   R

Um dia em uma reunião de família, surgiu a seguinte dúvida: Quando estamos lavando o carro
utilizando uma mangueira e tampamos o furo por onde sai a agua, o jato de agua sai mais fino e
mais rápido, ..... o que estamos aumentando? ?? A Pressão ? ?? A Velocidade? ?? Os Dois? ?? O
valor da pressão nao será sempre o mesmo?    R

ONDE POSSO OBTER INFORMAÇÕES SOBRE APLICAÇÃO DE LASERS NA MEDICINA?

Quantas dimensões existem comprovadamente na natureza, e quais são?   R

Quando um elemento radiativo decai, por quanto tempo sua eletrosfera consegue manter os
elétrons que ficaram sobrando?

Gostaria de saber se as iniciais das camadas da eletrosfera k,l,m,n,o,p e q tem algum sentido por
terem essas iniciais.. já procurei em todos os lugares e nao sei porque elas tem essas iniciais.
Quem as definiu?   R

(A) O que é uranio enriquecido? Qual a situação do Brasil em relação aos Estados Unidos? (B)
Como se enriquece o uranio? E qual sua finalidade apos ser enriquecido? O Brasil esta
enriquecendo uranio com qual finalidade?   R

Por que razão quando acendemos a boca do fogão, a chama só aparece por fora da boca. A
chama não poderia entrar no canal de gás, visto que ele é altamente inflamavel?   R

Quero saber sobre a propagação de som grave, som de contra baixo. Sou incomodado há 4 anos
por uma casa de som noturna de fins de semana. Já fizemas tudo o que foi possivel fazer e não
obtivemos resultados. O som começa as 22:00 e vai aaté 04:00 ~ 05:00 todas as sextas e
sábados. O que incomoda mais é a *batida* do som grave o tal de "tumtumtumtum". Moro numa
distância de aproximadaamente 70m da referida casa, não há nenhum obstáculo entre nós, o
som chega até o meu destino e de mais apenas 3 residências. A referida casa não possui
nenhum revestimento anti-acústico em suas paredes e teto. E não há meios de fazê-los se
adequar. A fiscalização faz medição com o decibelímetro mas, creio eu, que o decibelímetro não
capta o som de baixa frequencia. Quero a sua ajuda detalhada para que eu possa levar à
Prefeitura e, isso, sobre o som de baixa frequencia. O TUMTUMTUM é terríivel!   R

O que é afinal uma geodésica? (E em particular, uma geodésica do espaço-tempo?)   R

É verdade que as ondas eletro magnéticas continuam a viajar no espaço indefinidamente. E que
se uma nave captar nossa transmissão radiofônica ela poderá ser ouvida se existir um rádio
receptor em qualquer ponto do espaço? Ou seja, as transmissões feitas por rádio e televisão
poderão ser captadas por extra-terrestres inteligentes, se eles existirem?   R

Gostaria de saber como posso saber se um espelho possui algo por tras, ou seja, um espelho
falso. Certa vez me disseram que existe uma tecnica encostando a ponta de um lapis no espelho,
isso procede?   R

Ouvi um comentário de um amigo meu que dizia: Foi construída uma esfera (a mais perfeita
sobre a face da terra), que se fosse a Terra, sua elevação máxima seria de 2 metros. Essa esfera
será lançada aos cosmos para medir as influencias/interferencias da medição do tempo e os
efeitos que os movimentos de rotação e translação da terra provoca nesse processo. Há algum
fundamento nisso? Que, na verdade, está acontecendo nessa direção?   R

Li as respostas sobre a diferença entre potência sonora( PMPO e RMS ) que são atribuidas a alto-
falantes, amplificadores, mini system e etc. Elucidei a maioria das minhas dúvidas, mas gostaria
de saber se a afirmação que farei a seguir é verdadeira: "Em vários aparelhos de som cujas as
caracteristicas técnicas informam os dois níveis de potência, pmpo e rms, o que terá a melhor
qualidade sonora, será sempre o que apresentar a maior razão de RMS/PMPO."   R

Observamos ocorrência formações espiraladas nos mais diversos fenômenos, tais como: água
escorrendo por um ralo; tornados e furacões; buracos negros e galáxias elípticas. Por que certa
quantidade de matéria, sempre que é “obrigada”a “transitar” por um espaço restrito em período
relativamente curto de tempo, o faz segundo uma curva espiral?

Numa hipotetica rota de colisão entre 2 particulas massivas, Se os bosons fundamentais de uma
nao atingirem a outra, elas não se chocaram, uma passara pela outra como um fantasma e
durante um instante teremos 2 corpos num mesmo espaço?   R
Gostaria de saber quais são, em ordem, as cores que compões o espectro solar. Esta dúvida
surgiu durante uma palestra na qual o palestrante afirmou que, após a cor verde, vem a cor
laranja (enquanto eu penso que, a cor que se segue ao verde é a cor amarela).   R

Sou um jovem matemático(tenho quatorze anos) e gostaria de saber como são realizadas as
operações na álgebra dos octônions, quais são suas unidades, e também o que são números
ideais, introduzidos pelo matemático Kummer.Juntamente, quero saber se houve algum avanço
em pesquisas sobre divisibilidade por zero, porque tenho uma teoria admitível sobre o
assunto.   R

Como é possível, através de uma fibra ótica, visualizar e filmar o interior do corpo humano?   R

Quero saber como é o funcionamento do transdutor usado num sonar?

Compramos no meu predio um brinquedo chamado megaplay da mundo azul, e quando as


crianças escorregam ou pegam uma na outra levam choque, gostaria de saber se é normal, se é
prejudcial para a saude e qual a carga desse choque.

Um sistema não-linear é a mesma coisa que um sistema caótico deterministico?

Gostaria de saber o que é uma TEORIA COVARIANTE.   R

Tenho um colega que diz que por efeito da(s) Era(s) Glacial(ais), existe gelo embaixo da terra em
todo lugar! Disse ele que em Chapada dos Guimarães, MT, existe até gelo fossilizado! Isso é
intrigante! Como existe gelo em um lugar tão quente a uma profundidade considerável? E mais
interessante ainda: Gelo se fossiliza? Meu colega diz que geólogos já encontraram gelo
fossilizado, mas como hoje em dia existem muitos contadores de histórias, quero receber
resposta de alguém que realmente saiba!   R

O que é a interpretação de Copenhague?   R

Por que o calor especifico do gelo é menor do que o da água? O calor específico se relaciona de
alguma forma com as ligações intermoleculares?

Em qual ou em quais teorias e aplicações físicas posso encontrar a utilização de sistemas de


numeração de base diferente de 10?   R

Considerando que os buracos negros tem uma velocidade de escape maior do que c e a
velocidade dos hipoteticos gravitons e igual a c porque ele interage gravitacionalmente fora do
horizonte de eventos? Mais uma duvida, a teoria da relatividade preve que a luz sempre se move
com velocidade de c em relacao a qualquer referencial mas se a radiacao for emitida exatamente
sobre o horizonte de eventos ela estaria em repouso em relacao ao buraco negro. O que isso
significa?

Bom, uma é a questão de explosão no vácuo, "fogo" no vácuo. Sei que os filmes de ficção, são
exatamente FICÇÃO, mas se tratando de fogo no vácuo, seria possível? E a outra é como é
composto o fogo "artificial ou não", não sei, que se mantém dentro da água, como presenciamos
na abertura de uma das olímpiadas?   R

Quanta energia elétrica passa por um fio terra durante um mês em uma residência comum? É
possivel fazer uma estimativa?   R

Estive observando em um desses canais de televisão educativos, que a polaridade da Terra está
se invertendo... Existem documentações a respeito desse fenômeno? Que tipo de mudança seria
observada por nós durante a inversão?   R

O que seria um ciclone (tem algo a ver com maremoto?), e como ele é formado?   R

O princípio da incerteza de Heisenberg é demonstrado ou provado de alguma forma? Se for, qual


seria essa demonstração? O princípio de Heisenberg ou algum outro princípio relacionado à física
quântica impediria a construção de um algoritmo computacional que descrevesse a trajetória de
um elétron?   R
Gostaria de saber se existem outros combustíveis para automóveis não poluentes que não sejam
as células combustivel, o gás natural e como funcionam? Por que o combustível alcool não é
poluente?   R

O que é critério de RAYLEIGH?   R

Por que a maioria dos icebergs apresenta uma coloração em degradê azul?

Eu estava estudando uma teoria que dizia.... se a massa média do universo for maior que 10
elevado a -29 ele seria curvo negativo e infinito..... O q isso significa?   R

O que significa dizer que um operador temporal hamiltoniano é hermitiano?   R

É verdade que a Terra possui velocidade durante o dia diferente da velocidade durante a noite? Se
sim, qual o motivo para tal ocorrência?   R

Gostaria de saber qual é o princípio de funcionamento das lentes fotocromáticas.   R

Sobre o teorema da incompletude de kurt gödel cujo enunciado me foi dado como: "ele diz que
para todo sistema matematico formal e fechado criado por axiomas fundamentais depois de
certo grau de complexificação o sistema começa a "cuspir" respostas que não poderão ser
provadas NUNCA pelos que axiomas que fundamentaram essa matematica" gostaria que me
fosse dada uma resposta mais detalhada ja que é um teorema fascinante que desafia o bom
senso e logica e tambem um exmplo onde ele pode ser posto em pratica.   R

O que acontece se eu ligar um alto-falante com impedancia nominal de 8 ohms a um amplificador


de potencia com impedancia menor (2 ou 4 ohms por exemplo)?   R

Qual é a diferença entre polo geográfico e polo magnético?   R

Eu gostaria de saber algumas informações sobre Pangéia.   R

Por que as pilhas secas não são recarregaveis e qual a diferença que faz as pilhas recarregaveis
conseguirem carga?   R

Existe uma explicação física (não química) para que o sabão tire a sujeira da roupa?   R

O que aconteceria se um buraco negro se formasse na Via Láctea? E qual a possibilidade de isso
acontecer?   R

Eu li que os fisicos insistem em afirmar que o tempo nao flui,ele simplesmente e'.Cientistas
nunca descobriram qualquer coisa como um fluxo padrao de tempo na natureza.A passagem do
tempo e' provavelmente uma ilusao.Mas ai tem uma contradicao das teorias da relatividade,
porque o tempo e',o tempo em bloco,e tambem ele pode mudar sua taxa de fluidez,tanto com
campos gravitacionais,quanto se aproximando da veloc. da luz.O tempo entao nao flui,nao tem
padrao de fluxo, ele e', o tempo "congelado" que Paul Davies disse no livro dele, mas eu posso
sentir com um relogio o fluxo, ou a mudanca do fluxo dele,nessas experiencias de relatividade.
Como explicar isso?   R

Por gentileza, o que é sistema de loop aberto e sistema de loop fechado??? Estou lendo o livro O
PODER DA INTELIGENCIA EMOCIONAL, e fala sobro loop aberto e loop fechado, mas não
entendi.   R

Estava lendo as respostas de eletricidade estática e isso me despertou uma curiosidade. Onde
trabalho existe uma esteira e recentemente levei um choque quando eu passava uma caixa de
metal em cima da unica roldana de material plástico, o interessante é que esta esteira é muito
pouca usada. Quando esta roldana é utilizada ela acumula eletricidade estática, mas com o
tempo em desuso esta eletricidade não deveria se dissipar?   R

Sabe aquela experiência onde se pega uma caixa, faz um furo de um lado e do outro (por dentro)
coloca-se um papel branco e depois fecha a caixa deixando somente o buraco para que a luz
entre? Então, se fizer tudo certo, a imagem da luz que entra pelo buraco é formada no papel e
invertida. Eu queria saber porque se fizermos um buraco relativamente grande, a experiência não
funciona?   R

quais são as consequências do bóson de higgs existir, isso muda algum concento da física ou
algo assim? Qual é a diferença entre matéria escura e bóson de Higgs?   R

Qual a necessidade de um filtro protetor(de cor laranja) em aparelhos fotopolimerizadores de


odontologia que utilizam luz visível com comprimento de onda proximo de 470nm, se ao andar ao
sol tenho todas as freqüências do espectro visível e não uso proteção nenhuma? Seria a mesma
função de um par de óculos escuros?

Li em um livro que existem buracos negros "com mais de um" horizonte de eventos.Qual o
significado de mais de um horizonte? O que acontecerá a algo que ultrapace o primeiro
horizonte? e ao segundo? O que ocorre na regiâo intermediária aos dois horizontes? Essa região
possui nome?   R

Gostaria de saber se existe isolante para IMÃ. Gostaria de saber quais os materiais mais
utilizados e os mais eficientes. Exemplo: Tenho dois imãs, e tenho que separá-los com um
material de modo que eles não se grudem.   R

qual a historia da computaçao quantica? como funciona? para que serve? e quais os avanços
atingidos ate hoje?   R

Gostaria de saber se hoje já há estudos sobre uma possível volta no tempo, pois fiquei sabendo
que em nível quântico isso já é possível. O desvendamento do bóson de higgs faria alguma
diferença neesse sentido, que outros tipos de tecnologia poderia fazê-lo?   R

Eu gostaria de saber por que o nivel do oceano pacifico é maior do que o do oceano atlantico?

gostaria de saber o que são flutuações quânticas?   R

O que é um quasar?   R

Existe mesmo gravidade negativa? Ela é que faz o uinverso se expandir? Para onde o universo se
expande? Já se sabe o que há alem dos limites do universo?   R

É possível estimar o comprimento de onda (ou freqüência) das ondas gravitacionais? Que
equação poderia ser usada para a previsão? De que fatores depende a freqüência? Quais são as
boas fontes bibliográficas sobre o assunto (na Internet)?

A resposta da questão numero 0735 que trata da teoria futebolística do universo, de Jean-Pierre
Luminet, diz que ao chegar-se ao "fim" do dodecaedro você voltaria para o lado oposto do
dodecaedro. Gostaria de saber como eles chegaram a essa conclusão e como a ciência justifica
tal "retorno".   R

O efeito aharanov-bohm pode ser explicado classicamente? E qual é a melhor explicação


quântica?

Suponhamos fosse toda a possibilidade de aproveitamento do fenômeno da maré alta, em toda a


terra aproveitada para realização de trabalho, daria um valor absurdo de JOULES, não? Sem violar
a sagrada CONSERVAÇÃO, de onde viria essa energia toda? Isso altera o que nos corpos
envolvidos no fenômeno, visto que a maré alta esta agora produzindo trabalho (por exemplo, a
erosão)?   R

o que eh condensado fermionico?   R

Por que a água não é combustível? (Ou — o que dá no mesmo — a que condições deve satisfazer
uma substância, para ser considerada combustível?)   R

Uma hora de hoje é igual a uma hora de 30 anos atrás? Todos dizem que atualmente os dias
passam muito mais rapido que antigamente. Poderia o movimento do conjunto de astros da
galaxia ser maior e isto influenciar na duração da hora.   R
A que locais poderei recorrer de modo a esclarecer melhor a relação entre a teoria do caos e
arritmias cardiacas e/ou ataques epiléticos?

Por favor eu gostaria de saber coisas a respeito da mólecula SF6 - Fluoreto de enxofre, eu
gostaria de saber a geometria molecular dela, pois ja procurei em livros e varios sites e nao acho!
e sei que aqui vocs vao responder!   R

Ler em movimento (num onibus, por exemplo) pode provocar deslocamento da retina?   R

Qual a relação entre Redes Neurais e os Modelos Estatísticos/Probabilisticos?

Por que podemos morrer eletrocutado se um aparelho eletrodoméstico cair numa banheira onde
estamos, e não sofremos dano algum quando tomamos banho em chuveiro elétrico? Se
pensarmos bem, a resistência do chuveiro não tem isolamento, é desencapada e estamos em
contato direto com o chão. Não me parece diferente do caso de estarmos submersos em água.
Pois a situação é a de dois fios eletrizados (220 u 127v) submerso em uma *cuba* contendo
água.   R

Como é formada a chuva??? Qual é a diferença da chuva comum para a de granizo?   R

Como explicar, para alunos do ensino médio, o fato do elétron não colapsar (cair no núcleo) já
que sua carga e a do próton são iguais em módulo? (É preciso levar em conta a explicação
quântica do fenômeno)?   R

Queria saber o que é um espinor e qual sua aplicação.   R

Por que ao se rasgar uma folha fina de papel ela se enrola?   R

Quanto mais denso o liquido, mais rapidamente ele esfria? uma xicara de chá ou café demora
mais par esfriar do que a mesma quantidade de sopa cremosa, por exemplo. Isso é verdade? Por
que?   R

Os led's de infra vermelho alimentados com 1,5 volts usados para iluminaçao noturna para
filmagens *luz invisível* usados em portoes eletronicos e monitoramento de quaro de bebês
podem fazer algum mal a saúde? (olhos). Que tipo de onda estes led's emitem?   R

Quem foi q deu o ponta-pé inicial na lua para ela entrar no campo gravitacional da terra e
constantemente girar em torno dessa?   R

Qual a relação existente entre TEORIA DA RELATIVIDADE, TEORIA DAS CORDAS e UNIVERSOS
PARALELOS?   R

Estou precisando encontrar explicações, dentro do contexto da física para explicar, mais
tecnicamente o seguinte : como explicar que um corpo não pode está em dois lugares ao mesmo
tempo, principalmente em lugares diferentes e distantes um do outro. Preciso de uma resposta
que explique se isso é possível ou não e onde está amparada essa resposta.   R

Seria possível um extraterrestre chegar à Terra e ter a força equivalente à do Superman, se em


seu planeta a gravidade fosse muito superior à terrestre?? E ele seria capaz de dar grandes saltos
ou não ?? Seria resistente a tiros como é o Superman?? Se um ser humano fosse raptado e
levado para viver nesse planeta , seus descendentes desenvolveriam força superior à nossa?   R

Volta e meia se lê nos jornais que o relógio mais preciso do mundo acaba de ser construído, e
que ele erra 1 segundo em cada século (digamos). Mas, se ele é o relógio mais preciso do
mundo, como é possível saber quanto ele erra? Teria de ser comparado com um ainda mais
preciso, que não existe!   R

O tempo é dividido em horas, as horas em minutos, minutos em segundos etc. Se dividirmos o


tempo infinitamente, chegaremos a um "tempo mínimo"? Um espaço de tempo indivisível? Ou
dividiremos até desaparecer com o tempo, chegando ao vazio? Na primeira hipótese, o tempo
teria que ser formado por alguma coisa? E na segunda, se o dividirmos até acabar com ele, na
verdade o tempo não existe, não é mesmo? Perdoem-me se não consegui me expressar direito,
mas espero que possam me ajudar.    R
O que vem a ser "efeito borboleta"? Escutei isso de uma senhora que é formada em física, mas
não tive mais contato com ela.   R

Atualmente vejo muitos caminhões trafegando com o eixo suspenso. Sendo assim ele tem
menos pneus em contato com o pavimento e logicamente menor área de atrito no momento das
frenagens. Se esta frenagem ocorrer em piso molhado, entendo que a situação fica ainda mais
crítica. Vocês conseguiriam provar tecnicamente se este risco procede?   R

Tenho lido alguns autores e encontrado opiniões divergentes sobre o paradoxo de ZENÃO, um
dizendo que o problema não pertence à física e sim a matemática, outros dizendo que certo ramo
da matemática deu a solução para o paradoxo. Eu tentei solucionar o problema sem “AXIOMA”, e
dois objetos em MRU paralelos, um determinado ponto(ideal) naquele que tiver alguma dianteira
jamais é alcançado por um determinado ponto(ideal) daquele em desvantagem não importando
quanto esse seja mais rápido. Pergunto qual a explicação?   R

Na grande maioria dos livros de química, se diz que o próton é formado por 1 neutrino, 1 elétron e
1 nêutron, mas nos livros de física quântica, moderna e etc., o próton é descrito como formado
por 3 quarks: 2 quarks up, cada um com uma carga elétrica positiva de 2/3, e 1 quark down, com
uma carga negativa de 1/3. Afinal, o próton é constituído pelo quê?   R

O espaço e tempo são vistos como contínuos, mas a gravidade quântica em loop afirma o
contrário, mas quais seriam as diferenças observadas entre um universo contínuo e um
descontínuo no que tange ao movimento? Não seria o espaço-tempo descontínuo a solução para
o conhecido paradoxo de Zenão e o cálculo ( derivada e integral) inapropriados para dar um
tratamento matemático ao mundo físico na distância de Planck?   R

Só por curiosidade.... Qual o material usado nos captadores das guitarras e dos contrabaixos
elétricos? Ouvi alguma coisa a ver com "piezoelétrico"... Alguém saberia qual?   R

Quando acontece um relâmpago ou raio, são emitidas ondas eletromagnéticas em diversas


frequências, existe alguma relação entre a frequência em que é emitida a maior quantidade de
energia (diferente para cada raio) e a energia total do raio? Existe alguma fórmula para calcular
isto?

Por que a cor da centelha produzida pelo o Gerador de Van de Graaff quando outro objeto se
aproxima tem a cor azulada?   R

quero saber se os átomos que compõem os buracos negros estão em sua conformação normal
ou se TODAS as partículas estão "grudadas" umas nas outras,sem NENHUM espaço entre
elas,tanto dentro dos átomos(eletrosfera agregada ao núcleo)como entre átomos individuais?   R

Sabemos que sob a crosta terrestre estah repleto de magma, liquefeito, a altas temperaturas.
Minha pergunta eh: devido a que o magma eh tao quente? O que fornece a energia para mante-lo
a tao altas temperaturas?   R

O que são células combustíveis, quais as mais comuns e suas aplicações?   R

É sobre o avião de combate americano "Stealth" que, se diz, é invisível para o radar. Qual é a física
dessa "invisibilidade"?   R

Júpiter é um dos "gigantes gasosos" do sistema solar. Isto quer dizer que Júpiter é uma bola de
gás? Ouço também falar na atmosfera de Júpiter. Mas se êle é gasoso, a atmosfera não é ele
mesmo?   R

Sobre os anéis de Saturno. A pergunta é acadêmica, porque agora já se sabe exatamente do que
eles são feitos: a sonda viu! Mas, antes, vi num site que Maxwell tinha demonstrado que, por
questões de estabilidade, os anéis não podiam ser rígidos, mas sim formados por pequenos
pedaços, "calhaus", como dizia o site. Mas, no mesmo site, vi que um flúido possui uma forma de
movimento estável que é uma rotação rígida, em que o flúido gira com se fosse um disco. A
pergunta então é: antes de se verificar "ao vivo" não se podia pensar que os anéis de Saturno
fossem líquidos, em vez de uma "poeira de pedras"?   R
Se os átomos são formados por Protons, Neutrons e elétrons; os quarks se juntam para formar
os protons e os neutrons; o elétron é um lepton. Sendo assim, os leptons não se juntam para
formar algo?   R

gostaria de saber algo sobre radiação ionizante,seus efeitos sobre corpo humano,qual o meio de
proteção contra radiação,trabalhadores exposto a radiação,doenças causa sobre exposição,meio
de eliminar a exposição de um trabalhador.   R

Ola, gostaria de saber como esta a situação atual do sistema geografico brasileiro(SGP) e
tambem, o sistema de tempo do gps (Atômico, Dinâmico, Universal, Sideral e o tempo
coordenado- UTC)

Gostaria de saber o que sao "tardyons" e se existe uma palavra correspondente em português.
Esse termo eu vi em um livro de divulgaçao cientifica chamado "Simply Einstein", de Richard
Wolfson. Uma busca pelo Google atraves de páginas em portugues nao revelou uma unica
ocorrência.   R

O Instituto de Física da Unicamp se chama Gleb Wataghin. Quem foi esta pessoa e o que é que
ele fez para merecer esta homenagem? Algum político?   R

Porque as gaivotas voam em bando em forma de "V"?   R

Minha professora de quimica disse que o mercúrio não é tóxico nos estados líquido e sólido. Isso
é verdade?   R

Caros colegas, o fato do infra vermelho longo ativar a renovação celular, especialmente em
pessoas que já passaram por um processo de cancer e por meios de tratamentos químicos
conseguiram controlar a situação,a utilização do IVL nessas pessoas não poderia piorar o quadro
de saúde das mesmas por ativar as celulas cancerigenas?

Qual o nome da galaxia mais distante da terra?   R

Sabemos que o indice de refraçao de um determinado meio material depende da frequencia


luminosa, mas gostaria de saber se depende da densidade do material e que dependencia é essa.
Em muitos livros de fisica de segundo grau, diz que quanto maior a densidade de um liquido, por
exemplo, maior é o indice de refraçao. Uma inspeçao rápida em qualquer handbook de física
mostrará que essa afirmaçao é falsa. Afinal, existe ou nao essa dependencia com a
densidade?   R

Como funcionam as telas táteis dos caixas eletrônicos?   R

Por que a luz (radiação) azul (maior energia) penetra mais na água do que a luz vermelha (menor
energia)? Se for apenas por causa da quantidade de energia, porque a radiação UV, que possui
ainda mais energia que a radiação azul não penetra muito na água?   R

O que são bosões W?   R

Sabemos que existe a velocidade de escape para cada corpo. Um buraco negro, possui
velocidade de escape superior a velocidade da luz, logo não deixa nem a luz escapar. Agora eu
pergunto: se a velocidade de escape está relacionada diretamente a massa do corpo, como foi
possível haver a explosão do big-bang, se havia toda a massa do universo contida em um único
canto? Na explosão, a velocidade era superior a da luz?   R

Em relação ao efeito da temperatura sobre a condutância elétrica, no sistema nervoso humano, o


que prevalece?: O aumento da condutância por causa do aumento do metabolismo das células
nervosas ou a diminuição da condutância por conta do aumento da resistência, como ocorre em
qualquer corpo?   R

Eu fiquei sabendo que a agua possui varias funções e gostaria de saber um pouco sobre elas, são
elas: -sua função como termoreguladorado do meio ambiente -sua função no mecanismo de
absorção de energias pelas celulas -sua condutibilidade eletrica (seu PH) -sua influencia na
resistencia eletrica do corpo humano.   R
Porque se diz que quando se está na praia com o tempo meio nublabo a gente se queima
mais?   R

Lendo no site sobre o princípio de funcionamento das geladeiras a querosene me veio a seguinte
pergunta: Por que este princípio não é utilizado para refrigerar veículos de passageiros, já que os
motores térmicos liberam tanto calor para o meio ambiente?   R

Uma onda plana e monocromática de luz pode ser também descrita em termos de fótons. O que,
na linguagem dos fótons, distingue uma onda de polarização circular de uma onda de
polarização plana?   R

Hoje há muitos físicos trabalhando em instituições financeiras. Fui investigar numas homepages,
e descobri que eles usam, como instrumentos de trabalho, uma coisa chamada "martigale" e um
"equilíbrio de Nash". Do que se trata?   R

No Parque Nacional de Itatiaia existe um pico dificil de escalar que se chama, segundo o guia
local, Oquialine. Segundo ele, este homem era um cientista nuclear que viveu escondido no
Itatiaia, durante a guerra. Achei que era bobagem, mas, depois, descobri que houve um italiano na
USP chamado Occhialini. Será que a história é verdadeira? Este Occhialini foi um físico
importante, segundo o google.   R

Nunca entendi isso direito: por que os carros de tração dianteira derrapam, ao subir uma ladeira
molhada, mais facilmente que os de tração trazeira? E, neste caso, por que usar tração
dianteira?   R

Em algumas termas, como Caldas Novas, estado de São Paulo, a água jorra quente e abundante,
e tem a mesma temperatura no verão e no inverno. Qual é o processo de aquecimento?   R

Ouvi dizer que anti-sépticos bucais (ex.: Listerine, Colgate, etc) que possuem álcool etílico em sua
composição fazem com a placa bacteriana da boca fique mais "encrustada" do que antes, ou
seja, só servem para piorar a situação. Isso é verdade?

Alguém pode me explicar o que é o paradoxo do submarino, que foi resolvido recentemente pelo
físico brasileiro George Matsas?   R

Sou engº mecanico numa empresa de Fundição injectada, e gostaria de saber se é possivel
reaproveitar a energia termica que dispendo no fornos de fusão. Ou seja tenho 20 fornos todos
com aluminio liquido a ±650ºc fornos esses que libertam calor, mas o k eu keria mesmo era
colocar algo dentro (um possivel condutor) que pelo seu aquecimento devido a essa
temperaturas pudesse depois tranferir esses calor para algo k transforma-se essa energia
termica em energia electrica. uma das ideias também era aquecer agua em circulação para
depois criar um aquecimento central. mas a pergunta é esta: existe algum modo criar energia
electrica através da energia termica do forno.   R

porque os planetas gigantes não podem ter uma superfície sólida?qual é o limite superior de
tamanho que,teoricamente,um planeta rochoso poderia ter?   R

porque não se fazem microscópios ópticos com espelhos de aumento em lugar de lentes,como
nos telescópios de reflexão?   R

É sobre o conceito meteorológico de "umidade relativa". Vivi num país muito frio onde, no inverno,
não havia, ao ar livre, água líquida. A própria neblina, era formada por minúsculas partículas de
gelo. Nesta situação, a umidade relativa é o que? 0% ? Espero que algum meteorologista de
plantão me ajude!   R

GOSTARIA DE UMA SOLUÇÃO PARA ESTA PERGUNTA QUE VI NO SITE, SENDO QUE GOSTARIA DE
FILMAR A TELA DO MICROCOMPUTADOR DE MODO QUE ELA NÃO APRESENTASSE ESTES
PROBLEMAS: "Qual o fenômeno físico que explica a ocorrência de listas periódicas se
movimentando nos monitores de computador quando filmados por câmeras de vídeo? SERÁ QUE
EXISTE UMA FILMADORA ESPECIAL PARA FILMAR TELAS DE MICROCOMPUTADOR ? SE EXISTE,
QUAL É? E ONDE A COMPRO?   R
No livro de Rindler, "Essential Relativity", está escrito que, por causa da expansão do universo,
existem fótons que, embora "apontados" para nós, nunca nos atingirão. Basicamente todos os
livros que tratam de cosmologia dizem isso, e eu não estou pondo em dúvida. O que eu quero
entender é como este fato é compatível com a não-existência de movimentos com velocidade
superior à da luz. Pois se, qualquer que seja o tempo decorrido, o fóton nunca nos atingirá, isto
não quer dizer que a nossa velocidade é maior do que a do fóton?   R

A anestesia impede a condução de impulsos nervosos para o cérebro que acaba não percebendo
a dor, gostaria de saber como ela impede essa comunicação.   R

Poderiamos realizar um experimento com um pendulo de torção em uma nave espacial, em um


local onde a gravidade é nula? Que resultado teriamos para o periodo?   R

Eu queria saber se a pororoca é um soliton.   R

É absolutamente necessário que o ângulo da molécula seja 104,5 ° para a substância ser água? É
possível "ser" água com outro ângulo?   R

Por que numa superfície de um líquido, aparentemente tranqüila, os corpos que neste líquido
flutuam, tendem a ir para as bordas do recipiente que os contém? Seria algo que tem a ver com a
tensão superficial?   R

Uma estrela com a massa do Sol pode formar um buraco negro? Se a resposta é "sim", o Sol vai,
ou pode, virar um buraco negro?   R

Como algumas aves conseguem se orientar perfeitamente viajando longas distâncias. Ouvi dizer
que tem algo a ver com o magnetismo da Terra. Mas qual a ligação entre a ave e o magnetismo?
Elas só podem viajar entre Norte e Sul, ou entre Leste e Oeste e outras direções também, em
longas viagens?   R

Eu tenho muito interesse por física de partículas e queria saber exatamente quantas partículas
elementares o homem já descobriu até hoje (todas elas se vc puder me responder).   R

Como podemos descobrir se o núcleo de um planeta é pastoso (no caso da Terra) ou solidificado
(no caso da Lua)? Existe alguma outra maneira que não seja através da detecção da existência
de campo magnético no mesmo? E, para terminar, existe a possibilidade do campo magnético ser
gerado por outra razão que não seja pelo movimento das cargar elétricas devido a convecção dos
metais constituintes no núcleo?

1 - Como que se dá a utilização do método de Monte Carlo no estudo nos chuveiros


atmosfericos, consequentemente dos raios cósmicos? 2 - Onde encontro boa literatura sobre o
assunto?

Sabemos q existe limite inferiores para uma temperatura.No entanto, nao existe um limite
superior. Se a temperatura é a medida da energia cinetica, quanto maior é a temperatura, maior é
a velocidade das moleculas do gás. Mas a T da relatividade poe um limite para as
velocidades.Entao, nao deveria existir uma temperatura limite para uma determinada massa de
gás confinada em um recipiente qualquer?   R

Curiosidade. Todos já ouviram falar das famosas máquinas "centrífugas", utilizadas para separar
soluções ondes os materiais dissolvidos têm densidades diferentes. Os materiais mais densos
vão para as posições de maior raio na trajetória. A curiosidade é: vocês já colocaram uma colher
de açucar no chá e mexeram? O açucar não dissolvido vai para o CENTRO da xícara, e não para
as bordas, mesmo se bem agitado junto com o chá. Como explicar isto dentro das leis da física?
Que lei rege este fenômeno? Ele está em contradição com o caso das centrífugas?   R

Fala-se q a temperatura é a medida da energia cinética média das moleculas. Sendo assim, como
se mede a temperatura , ou qual é o parâmetro para medir a temperatura na Lua? Sabemos q na
Lua as temperaturas variam muito quando bate o sol e quando nao bate o sol em um
determinado ponto. Como se sabe esses valores então?   R

Olá pessoal, gostaria de saber como fazer o cálculo da força elétrica gerada por um vento que
sopra em um moinho, sabendo a velocidade do vento, a temperatura e pressão locais e o
diametro das pás do moinho!   R

Alguns animais são capazes de perceber sons de freqüências exteriores a faixa audível para o
ouvido humano. Os golfinhos e os morcegos, por exemplo conseguem ouvir ultra-sons com
freqüências seis vezes maiores que o limite de audição humana. Como o golfinho e o morcego
conseguem se movimentar tirando partido desse fato?   R

Há alguma razão física para que a frequencia modulada das rádios terminem sempre em número
ímpar?   R

Os medidores de ruído do mercado, decibelimetros conseguem medir o som de bate-estaca de


uma boate? Porque vi nas especifições técnicas de alguns que a frequencia minima sensível é
10hz ou 25hz. O som de bate-estaca da boate a duas quadras (moro em zona rural) chega bem
aplificado ao meu quarto. Se eles não conseguem, então não servem para medir este tipo de
poluição sonora.   R

Li em um livro que segundo a Teoria das Histórias de Feynman todas as histórias existem,
inclusive para corpos macroscópicos. porém parece que não, devido a cancelamento de
histórias.Como acontece esse cancelamento? existe uma história onde eu ganhei na mega sena
e não enviei essa pergunta? Qual o significado dessa Teoria de Feynman? Para que servem as
equações de Tomonaga-Schwinger? Por favor me ajudem ou vou morrer de curiosidade!   R

A velocidade-da-luz é propriedade da luz (onda eletromagnética) ou do espaço no qual se


propaga a luz? Se a velocidade da luz muda de meio para meio, por que não acreditar que essa
velocidade seja um limite do espaço-tempo e não da luz em si?   R

Como é possível fazer um filtro para atenuar interferências de RF nos canais de TV? Sei que
tenho que utilizar alguns capacitores cerâmicos, varistores, mas como se dá a associação para
filtrar o que é desnecessário?   R

No lançamento de um desses gigantescos foguetes no Cabo Canaveral, no início, aquela enorme


estrutura esgue-se muito lentamente, para depois ir ganhando uma enorme velocidade. Mas, no
começo, quando ainda está bem lento, que tipo de mecanismo o mantém rigorosamente na
vertical?   R

Estudamos nos cursos elementares de mecânica quântica o espectro do átomo de hidrogênio.


Seus níveis de energia, proporcionais a 1/(n quadrado), para n > 0, são bem conhecidos. Mas o
gás hidrogênio é formado de moléculas biatômicas, cujo espectro é diferente. Como se faz para
testar empiricamente a fórmula do espectro do hidrogênio atômico?   R

Do que realmente era formada a sopa primordial?   R

Como surgiu a água liquida em nosso planeta?   R

Qual a pronúncia certa da palavra "Joule" e pq tem esta pronúncia?   R

He estado intentando investigar sobre la Física que rije el cambio de trayectoria en una
moto/bicicleta, sin llegar a una conclusión sobre si todo se reduce a una descomposición de
fuerzas al inclinar la moto, o interviene también el cambio de radio de la rueda al inclinarse, o si
finalmente es imperiosa la necesidad de girar, aunque sea algo, el manillar. Si existe alguna
publicación al respecto o tienen alguna respuesta se las agradeceré.

Como funciona um fabricador de neblima? (Calma, vou descrever). É uma peças com led's e é
colocado na agua e quando você liga começa a fazer uma neblima artificial. Vocês poderiam me
explicar como isso funciona?   R

Em minha leiga opinião, sentimos sede porque nosso organismo se desidrata e precisamos repor
essa água perdida. Minha pergunta é: por que sentimos uma necessidade quase que urgente de
beber água ao ingerirmos alimentos extremamente doces ou extremamene salgados? Eles
provocariam algum tipo de desidração instantânea?   R

Existem circuitos que permitem acender uma lâmpada, por exemplo, em um local e desligá-la no
outro, e vice-versa, usando dois switches. Para isso, três fios elétricos são necessários. Tenho um
problema no qual posso utilizar apenas dois fios e ainda assim permitir ligar e desligar a lâmpada
em dois locais diferentes. Fiquei sabendo que exitem switches interligados, você liga um deles e
o outro é ligado automaticamente, ao mesmo tempo. Queria saber se existe mesmo e como esse
tipo de switch funciona.   R

Presto serviços de Closed Caption, em "Tempo Real", para surdos. Hoje os surdos solicitam
legenda nos teatros, contudo, a legenda em telões ou com display de leds, pertuba os ouvintes.
Gostaria de saber se temos no país dislpays de 8 segmentos entre 30 ~50 cm de altura, para que
pudessemos colocar no "Pé do Palco" sem o filme polarizador, dessa forma ninguem veria o que
estava sendo escrito, exceto quen tivesse o óculos com o filme, no caso os surdos. É possivel?
Desde já agradeço Obs. Temos 7 milhões de surdos no Brasil.

Segundo Stephen Hawking próximo ao horizonte de eventos de um buraco negro podem surgir
pares de partículas e anti-partículas, um dos membros desse par pode cair no buraco negro
enquanto a outra fica livre para escapar. Essas partículas que escapam são interpretadas como a
chamada Radiação de Hawking que leva embora a massa do buraco negro. Agora vem as
perguntas: esses pares de partículas são criadas dentro ou fora do horizonte? Se for dentro,
como conseguem escapar? Se for fora, porque o buraco negro perde massa, já que as partículas
que escapam não pertenciam ao buraco negro?   R

Tenho um filtro de barro de 08 litros, ele esta proximo a janela da cozinha, porem não recebe luz
direta do sol, mas, recebe a sua claridade. Porém por fora do filtro estão aparecendo pigmentos
de mofo, hoje eu levantei a parte de cima para verificar e notei que a parte de baixo (do lado de
dentro) estão se alastrando o mesmo mofo que estão fora. O que devo fazer para evitar isso???
Preciso mudar de lugar??? Se colocar em outro comodo da casa (cozinha 2) onde nem mesmo
entra luz solar, vai resolver? Preciso "banhar" com algúm produto químico ou natural?   R

Gostaria de saber o que são umas linhas de fumaça que se forma ao lado do cogumelo,segundos
após uma detonação de uma bomba nuclear em solo.   R

Onde posso comprar filme ou plástico polaróide para fazer um óculos 3D para feira de ciências
(já que os filtros de máquinas fotográficas são caros)?

O equipamento que trabalho gera energia estática ininterrupta e não consigo descarrega-la
somente com aterramento. Como transforma-la em corrente continua para ser consumida por
uma lampada, por exemplo?

Ficaria muito grato se alguem REALMENTE conseguiria responder essa pergunta. Em vários
livros de física diz que quanto maior é a profundidade, maior é a velocidade da onda na água.
Nunca encontrei o porque disso em livro nenhum. Isso sempre é verdade? Tem exceçao? Qual é a
equaçao q governaria o movimento?   R

gostaria de saber quais as possibilidades de acontecer Furacões e tornados no Brasil? O que


estimularia isso a acontecer?   R

Quais as diferenças básicas entre a teoria evolucionista defendida por Darwin e o criacionismo,
uma nova teoria defendida por vários cientistas ao redor do mundo. Gostaria que as respostas
fossem publicadas.   R

Gostaria de saber se é verdade que quando acordamos, logo no início do dia, nossa altura é
maior que ao final do dia... isso é devido a força da gravidade ? em caso positivo, alguém poderia
me informar se existe algum site em Ingles com respaldo científico de que essa máxima é
verdadeira?   R

Queria saber alguma coisa a respeito da Teoria dos Grandes Numeros. Eh q meu professor de
fisica soh mencionou na aula e naum explicou...e eu fiquei seca pra saber mais alguma
coisa.Alguem pode me ajudar?   R

Gostaria de saber um pouco mais sobre polarição de antenas, eu sei que a polarização da antena
acompanha a direção do campo elétrico, portanto no caso de uma antena do tipo Yagi, o campo
elétrico está no mesmo sentido dos elementos da antena, ou seja se os elementos estão na
vertical a polarização é vertical. Mas eu já ouvi dizer que no caso de antena parabólicas de link de
microondas, a polarização da antena é normal à posição do "cabo de guarda-chuva", porque isso,
na realidade o campo elétrico está normal ao "cabo de guarda-chuva"?   R

O calor latente de vaporizaçao da água é de 540 cal/g .Entao fica claro q se eu quiser passar 10
gramas de água para vapor teria q fornecer 5400 cal só para mudar de fase. Mas e se o processo
for de evaporaçao e nao de ebuliçao?? Como a evaporaçao ocorre a qualquer temperatura, a
energia necessária para evaporar 10 grams de água seria tb de 5400 calorias?? Caso contrario,
de quanto seria? E se pensarmos em termos de sublimaçao de uma substacia?? A energia
necessária para sublimar seria a soma do calor latente de fusao mais o de vaporizaçao
(considerando é claro q a pressao sob o material está abaixo da pressao do ponto triplo)?   R

Fala-se muito em aceleradores de particulas q conseguem altas energias , da ordem de Mev ou


Gev. O q eu acho estranho é q essas energias em joule sao extremamente baixas. Ora, pq nao dar
uma pancada com um taco de beisebol em uma particula?? Com certeza eu nao acertaria uma
só, mas se tiver um monte delas, elas receberiam energias de muito joules. O q está errado no
raciocínio?   R

Olá! Numa fonte artesanal (dessas usadas em jardins), vi um aparelho capaz de produzir uma
espécie de fumaça (névoa) que gerava um efeito muito interessante. Pesquisando um pouco,
descobri que se tratava de um circuito eletrônico (um oscilador de 1,6Mhz com cristal
cerâmico)que quando submerso na fonte é capaz de desorientar as moléculas da água e gerar
esse efeito. Gostaria de uma ajuda para saber se minha pesquisa tem algum fundamento, qual o
fenômeno físico envolvido nesse processo, e se alguém teria um diagrama esquemático desse
circuito. (Laudos médicos, dizem que essa fumaça ou névoa gerada apenas pela vibração na
água, é muito benéfica para a saúde quando a umidade relativa do ar está baixa, principalmente
para as crianças com problemas respiratórios.) Se alguém puder me ajudar, agradeço de montão.
Um grande abraço à todos.   R

Sobre a resposta dada ao tópico de Boosters X Mosfet: se o mosfet é um amplificador feito a


partir da elevação de tensão, então se eu pegasse um projeto de amplificador qualquer potente, e
construisse um inversor, eu poderia então construir um amplificador de mesma potência e
qualidade que um Mosfet?   R

Moro na Alemanha e estou interessada em comprar uma televisao aqui, mas só encontro 50HZ....
bem, ouvi dizer que nao tem problema, pois nao tem motor e nem iluminacao.... e que omonitor
50 Hz também funciona aí no Brasil. Gostaria de saber se é verdade, desde já agradeco.   R

O "aleatório" existe? Qual a definição de aleatoriedade, já que fisicamente se sabemos as


condições iniciais de um sistema poderemos prever sua configuração final? Algo que
teoricamente pode ser reproduzido não deixaria de ser aleatório?   R

Eu queria saber sobre a questão das velocidades dos planetas.No inicio os planetas giravam
mais rápido que atualmente,e sua tendencia no futuro é parar.Como esse movimento influenciou
o desenvolvimento dos seres vivos e como nos afetará?   R

O que é e como se forma um fogo de santelmo?   R

Revisitando a pergunta do Tuiã (qr0250), na qual ele pergunta o que aconteceria se uma partícula
de matéria e uma de anti-matéria, mas que não fossem anti-partículas entre si, fossem colididas.
Peguemos, para exemplo, um próton e um anti-nêutron. O próton é composto de três quarks (dois
up e um down). O anti-nêutron é composto por três anti-quarks (um anti-up e dois anti-down). O
que impede que um quark up colida com o anti-up e o down com um dos anti-down? E que raios
daria o quark up e o anti-down restantes?   R

Olá, eu gostaria de saber como é fisicamente o funcionamento do marcapasso cardíaco , o


circuito eletronico, do que ele é composto e entender por que as microondas causam
incompatibilidade eletromagnética nesse tipo de aparelho podendo acarretar problemas para
quem é portador.O que gera a interferencia e o problema de pessoas portadoras de marcapasso
utilizarem celulares e forno de microondas entre outros.   R

- desde 1957 que estudo, trabalho, divirto, pesquiso e invento com eletrônica mas, até hoje ainda
não consegui descobrir o motivo e a razão do valores padronizados para os componentes
resistor, capacitor e indutor, a saber: ( 1X, 1.2X, 1.5X, 1.8X, 2.2X, 2.8X, 3.3X, 4.7X, 5.6X, 6.8X e 8.2X
) onde "X" pode ser a unidade, kilo ou mega. - help!   R

A vida está surgindo atualmente? Isto é, está ocorrendo, hoje em dia, a formação de coacervados
e proteínas nos mares?   R

Se é verdade que se houver a inversão do fio neutro com uma fase na bobina de corrente nos
terminais de medição (medidor de energia), o medidor passará a registrar consumo irreal. Se isso
realmente ocorre, quando será registrado para mais (à favor da companhia de energia) e quando
sera registrado para menos (a favor do usuário. Considerar uma ligação trifásica. Obrigado.   R

O princípio de Pascal fala da transmissao do aumento de pressao em um fluido. Vamos


considerar que temos um copo com metade de água embaixo e metade de óleo em cima. Se
tamparmos o recipiente hermeticamente e pressionarmos a tampa, haverá um aumento de
pressao na superficie do liquido e q será transmitido para todo o léquido. Minha pergunta é a
seguinte: Mesmo ao mudar para um liquido mais denso, o acrescimo de pressao será o mesmo?
O priinc. de Pascal é válido tb para misturas heterogeneas?? A propósito: Do ponto de vista
microscópico, como ocorre esse aumento de pressao? Que eu saiba um líquido é praticamente
incompressível.

Queria saber mais sobre o que vi em um filme, acho que se chama fogo dos gregos. um fogo
vindo de substancias quimicas ( pó), que quanto mais agua se joga mais ele cresce e se
espalha.   R

Existe mesmo a bomba magnetica ou medusa magnetica, se existe como funcionam? Dizem q
elas nao afetam os seres vivos mais destroem todos os eletro eletronicos. É verdade?   R

A quantidade de movimento é derivada da energia cinética, certo? Pois se tomarmos um corpo


com velocidade v próximo a c, essas equações não valem mais, e entram as de Einstein. Então,
se eu integrar a equação da quantidade de movimento relativística Qr = mv/[1-(c/v)2]^1/2 eu
posso obter uma equação para a energia cinética relativística?   R

Quando trabalhei há mais ou menos 20 anos numa multinacional americana da área de


informática havia um componente eletrônico que submetido a uma corrente elétrica resfriava ao
invés de dissipar calor. na época achei que seria o futuro dos condicionadores de ar, com placas
maiores evidentemente. gostaria de saber qual é o fenômeno físico que estava ocorrendo e que
tipo de material semicondutor seria o utilizado naquele componente.   R

Vi a resposta que vocês deram sobre o microscópio de tunelamento. Pergunto o seguinte:aquelas


imagens obtidas com o microscópio, muito divulgadas, em que se vê uma superfície com
saliências como se fossem "montículos", que representa? Os montículos são átomos? Núcleos
dos átomos?   R

Fiquei sabendo que os motoristas de caminhão costumam colocar alguns CD's no parabrisa e
também na parte traseira da carroceria do caminhão. O motivo para isso é que o CD impede que
o radar da Polícia Rodoviária "veja" o veículo e que as câmeras que fotografam veículos com
excesso de velocidade também não são acionadas devido ao CD. É possivel obter-se este efeito
com o uso de CD's nos veículos?   R

Uma célula foto-elétrica converte ondas eletromagnéticas em corrente elétrica As ondas de rádio
são eletromagnéticas então a questão é: É possível converter ondas de rádio numa corrente
elétrica fraca e usa-la para acender pelo menos um diôdo luminoso? ou quem sabe mover um
leve carrinho de brinquedo? E se for produzido ondas de rádio de alta frequência é possível mover
um carro maior? Estou pensando assim: Quanto maior a frequência maior a energia da onda logo
maior será a corrente elétrica.   R

De acordo com o que estudei, o meio no qual o corpo está imerso (ar ou líquido) oferece também
uma resistência ao deslocamento. Por exemplo: um corpo abandonado do alto de um prédio
adquire movimento acelerado por causa da ação da força peso. Além dessa força, atua no corpo
a força de resistência do ar, que tem a mesma direção e sentido contrário ao da força peso. Essa
força de resitência do ar é variável e depende da velocidade do corpo, de sua forma e da maior
secção tranversal em relação à direção do movimento. Pois bem, poderiam explicar-me,
detalhadamente por que uma gota de chuva cuja velocidade é de 2 m/s, a força de resistência do
ar é proporcional a essa velocidade? E por que os corpos pequenos cuja velocidade varia entre 24
m/s e 330 m/s, a força de resistência do ar é proporcional ao quadrado da velocidade?   R

Alguém sabe me dizer por que todos os livros de físico-quimica apresentam sem prova ou
demonstração a equação de energia livre de Gibbs? Alguém saberia demonstrá-la?   R

O fato da água ter ligações do tipo "pontes de hidrogênio", faz com que ela solubilize mais fácil o
soluto? Ex.: tenho um complexo polar, e verifiquei experimentalmente que ele foi solúvel em água
(que é polar), e nao foi em metanol, etanol, entre outros que também são polares. Porque isso
ocorre?   R

Eu entendi como se formam os elementos químicos nas apostila da dona Fifi. Minha dúvidade é a
seguinte. De onde vieram os elementos químicos q estao em nosso planeta?? Se a terra foi um
pedaço do sol (dizem alguns), entao nao poderíamos ter elementos acima do Berílio. De onde
veio o resto entao?? De onde veio o plutonio, uranio e outros elementos pesados q estao
presentes aqui na Terra??Nao me entendam mal.....já sei como os elementos se formaram nas
estrelas....quero saber como eles chegaram aqui na Terra!!!   R

Quando um elétron desce um nível de energia em um átomo, a energia se transforma em um


fóton. Todos os fótons viajam na velocidade da luz. Velocidade tem uma direção. O que
determina a direção em que o fóton irá sair do átomo quando um elétron cai um nível de
energia?   R

Por que a Ag, Cu e Au são considerados metais diamagnéticos se eles apresentam 1 elétron
desemparelhado? Ag com Z = 47 ( Kr )4d10 5s1 Cu com Z = 29 ( Ar )3d10 4s1 Au com z = 79 ( Xe
)4f14 5d10 6s1.   R

No filme recém lançado "O Núcleo", um geofísico explica que a formação do campo
eletromagnético natural da Terra se deve pelo movimento de convecção do núcelo do planeta,
que é formado por metal líquido em altíssima temperatura. Assim como a Terra, todos os outros
planetas também possuem um campo eletromagnético?

O que é o efeito corona?   R

Até onde sei, distinguimos a posição de uma fonte sonora em relação ao ouvinte pela diferença
do tempo em que o som chega a cada ouvido. Isto é, se a origem do som está a direita, o
percurso deste som até o ouvido direito é menor do que em relação ao ouvido esquerdo. Assim
nosso cérebro detecta esta pequena diferença, para calcular a posição. No entanto, como
distinguimos se um som vem da frente ou de trás? Sistemas de home theater utilizam 5 ou mais
caixas acusticas para distribuir o som no ambiente. Se só temos dois ouvidos para receber este
som, não seria possível reproduzir um som multicanal a partir apenas de duas caixas?   R

É verdade que um elétron pode estar em dois lugares ao mesmo tempo sem se dividir?   R

É verdade que o corpo humano produz mais bioeletricidade do que uma bateria de 120V como é
dito por Morpheus, personagem da série Matrix?   R

Por que o mercúrio é líquido? Não vale responder que é devido ao seu baixo ponto de fusão. A
ligação metálica prevê uma interação interatômica muito intensa, então, pq mesmo assim, o Hg é
líquido à temperatura ambiente?   R

Gostaria de saber onde encontrar (se existir) fabricantes de turbinas a gás alimentadas a
hidrogénio(hidrogénio este produzido por paineis solares).   R

Existem máquinas que se sabe que não é viável desligá-la se esta vai ser usada em breve, por
exemplo, um ferro de passar, que perde calor e a energia usada para "reesquentá-lo" é grande. A
minha dúvida é: quais máquinas compensam ou não ser desligadas? Não o tipo de máquina
exatamente, mas em relação a sua potência, a partir de que valores passa a compensar desligar
ou não a máquina?   R

Por que a força forte, fraca e eletromagnetica sao atrativas e repulsivas, e a força da gravidade é
apenas atrativa? Existe a antigravidade?   R
Sou Professor da UFMG trabalhando na área biológica (Patologia). Em uma das minhas linhas de
pesquisa tenho estudado a chamada inflamação granulomatosa. Nessa patologia as células
inflamatórias (leucócitos) formam estruturas redondas, esféricas ou mesmo elipsóides camadas
granulomas em diversos órgãos. De acordo com as teorias e ideías da física já li e ouvi dizer que
a forma circular resulta em menor gasto de energia das moléculas ou corpos. Já li também nesse
site por exemplo, o porque dos planetas serem redondos. Pergunta: Gostaria saber ou obter
maiores informações ou orientações de como posso associar (se existe referencias científicas
que pudessem me ajudar) o porque da formação dos granulomas e as leis da física.   R

Meus amigos e eu entramos numa contenda em que as opiniões se dividiram a respeito do


tempo de cozimento numa panela de pressão. Eu defendia a posição que depois que uma panela
de pressão começa a liberar o execesso de pressão, pode-se baixar o fogo, pois a válvula
mantém a pressão constante e por conseguinte o ponto de ebulição da água será também
constante. Desta forma o alimento ficará exposto a pressão e temperatura constantes. Mais calor
na panela implicará apena em mais agua se transformando em vapor, pois a energia que não
aumenta a temperatura da água irá modificar seu estado físico... Essa é minha opinião, mas o
argumento que não concordei é que desta forma seria possível tocar uma locomotiva com uma
vela. Disse que não pois o que move a locomotiva é exatamente o aumento da pressão e a agua
se transformando em vapor é que movimenta os pistões, e desta forma quanto mais fogo, mais
pressão e mais expansão da agua... Outro argumento é que uma quantidade maior de calor
"passaria" pelos alimentos com a panela de pressão no fogo alto, mas argumentei que essa
"passagem" teria efeito desprezível na medida que a pressão e a temperatura ainda estaria
constante... Pode me esclarecer?

Quando aquecemos um pedaço de ferro até ficar rubro ele passará a emitir fótons. Pelo que
entendi sobre a teoria das cordas, alguma coisa fará com que cordas, que formam o tecido
espaço-tempo, vibrem em um determinado padrão para que sejam "gerados" as partículas
"fótons". Quando elétrons são exitados indo para níveis quânticos mais elevados, ao caírem a
níveis quânticos mais baixos perdem a energia que receberam. A pergunta é: Como é feito, de
acordo com a teoria das cordas, essa conversão de energia perdida, durante o decaimento
quântico, em fótons?

Olá, certa vez eu li (não me lembro a fonte) que até um período da história humana, a contagem
era feita por algarismos romanos e, após um certo tempo, passaram a usar os algarismos
arábicos, convertendo os romanos para arábicos. Nessa mudança, houve uma somatória errada
e, com isso, teríamos perdido, não me lembro exatamente, entre 5 e 7 anos e, na realidade,
estaríamos entre 2010 e 2012. Isso é verdade? Além disso, alguns dizem que Cristo não teria
nascido no dia 25 de dezembro do ano 1 e sim entre março e abril deste mesmo ano. Vocês
poderiam esclarecer esta dúvida?   R

O modelo padrao afirma que as interacoes fundamentais acontecem via troca de particulas,seja
fotons,gluons,particulas z,w.Agora,existem varios fisicos que respondem dizendo que as
particulas de forca(ou de gauge)NAO SAO VIRTUAIS,pois sao detectadas.Agora,existem varios
sites que afirmam isso(que sao virtuais) como o da Britannica,onde se fala- > " Finally, the picture
of electromagnetic interactions as the exchange of virtual particles has been carried over to the
theories of the strong, weak, and gravitational forces. "Entao essas particulas sao ou nao virtuais?
E se nao forem, a explicacao para esse modelo padrao viola as leis de conservacao.Para citar um
exemplo do decaimento do neutron, que ocorre quando um dos quarks do neutron emite um
boson W carregado e entao decai em um eletron e um antineutrino.O boson w tem 80 vezes a
massa do neutron,entao se ele for real,a medicao dele violara a conservacao de energia.    R

Segundo a teoria da relatividade, quanto mais denso for um corpo maior será a deformação que
ele causará na estrutura espaço-tempo, ou seja, à medida que me aproximo de um corpo denso
emitindo luz para um observador que se encontra parado atrás de mim, a luz piscará numa
frequência menor para o observador do que para mim, uma vez que o tempo para mim passará
mais rápido do que para o observador que está parado e afastado, longe da deformação na
estrutura espaço-tempo, criada pelo corpo denso (um planeta por exmplo). A dúvida é a seguinte:
Considere um sistema formado pelo sol, terra e lua. Como pode o tempo ser diferente para cada
um dos três elementos do sistema já que o sistema é um só? O tempo medido para a luz vir do
sol até a terra seria diferente de 7 minutos se medido no sol ou na lua?

Você também pode gostar